You are on page 1of 75

Acid-Base

You are evaluating a 2 year old female in regard to allegations of sexual contact that occurred while in the care of her stepfather. On
examination, you note normal external female genitalia and an intact hymen.
Which of the following infections, if present, would be most supportive of sexual abuse?
1) Candida albicans
2) Herpes simplex virus (HSV)
3) Human papillomavirus (HPV)
4) Neisseria gonorrhoeae
5) Trichomonas
The correct answer is : D.

Allegations of sexual abuse can be difficult to support medically. In such cases, the history provided by the child and the adults involved
is usually the most useful in supporting or excluding whether or not the abuse did take place, but medical professionals are frequently
asked to assess for the presence of physical signs of abuse. Additionally, on occasion, laboratory findings can be helpful in providing
evidence that sexual contact occurred.

When testing for infections that can be sexually transmitted, it is important to note that such infections can often be transmitted in other
ways. Thus, their presence is not necessarily pathognomonic for abuse.

The presence of some infections, however, is highly specific for sexual abuse, including gonorrhea, chlamydia, syphilis, and HIV. Even
for these infections, however, it is extremely important to remember that perinatal transmission can occur - testing the mother, when
possible, can help to support or refute that possibility.

Candida albicans (choice A) can occur in patients who have had no sexual contact; its presence does not support a diagnosis of sexual
abuse.

Herpes simplex virus (choice B) is transmitted by close contact that need not be sexual in nature. While HSV may support a diagnosis
of sexual abuse, this infection can also be transmitted in more casual ways.

Human papillomavirus (choice C) can also be transmitted non-sexually. Additionally, it can be transmitted perinatally by mothers who
have no overt history of condyloma acuminatum but who have a history of abnormal Pap smears; the HPV in the cervix can cause
infection in the infant, which may take months to years to present as discrete warts.

Trichomonas (choice E) infection is suggestive of sexual abuse but can also occur in cases where no sexual contact has occurred.

A 58-year-old woman with a history of epigastric tenderness and diarrhea is found to have a peptic ulcer and hypertrophic gastric fold on
a upper GI series. A plain film of the abdomen is obtained.
Which of the following would correctly describe her acid-base status?

1) metabolic acidosis

2)
3)
4)
5)

metabolic alkalosis
respiratory acidosis
respiratory alkalosis
mixed acid-base disorder

The correct answer is : B

The prominent gastric air bubble, gastric air-fluid level, and a dilated stomach with particulate matter within it shows evidence of gastric
outlet obstruction. Gastric outlet obstruction is an obstruction at the level of the pylorus, which is the outlet of the stomach. Individuals
with gastric outlet obstruction will often have recurrent vomiting of food that has accumulated in the stomach, but which cannot pass into
the small intestine due to the obstruction. The stomach often dilates to accommodate food intake and secretions. Causes of gastric
outlet obstruction include both benign causes (such as peptic ulcer disease affecting the area around the pylorus), as well as malignant
causes, such as gastric cancer. Treatment of the condition depends upon the underlying cause; it can involve endoscopic therapies
(such as dilation of the obstruction with balloons or the placement of metallic stents), other medical therapies, or surgery to resolve the
obstruction.

As a result of the obstruction, the patient has chloride-responsive (<10 mEq/L) metabolic alkalosis. In chloride-responsive metabolic
alkalosis, alkalosis can result from loss of hydrogen ions from either vomitting (direct loss of hydrogen and chloride ions with stomach
contents) or a contraction alkalosis that develops from from the loss of water in the extracellular space which is poor in bicarbonate.
Since water is lost while bicarbonate is retained, the increased concentration of bicarbonate "mops up" more of the hydrogen ions and
raises the blood pH. Hydration with normal saline corrects chloride-responsive metabolic alkalosis.

Metabolic acidosis (choice A) would arise in situations such as diarrhea (non gap acidosis) or the MUDPILES high anion gap acidosis
(methanol intoxication, uremia, diabetic ketoacidosis, para-aldehyde, isoniazid/iron, lactic acid, ethyleneglycol, salicylates).

Respiratory acidosis (choice C) would result from decreased respiration (hypoventilation) and increased PaCO2 (hypercapnia).
Respiratory acidosis can result from problems with the lung (COPD or ARDS) or problems outside the lung that impairs respiration as in
the case of neuromuscular disorders (e.g. Myasthenia gravis).

Respiratory alkalosis (choice D) would result from increased respiration (hyperventilation) that elevates the blood pH. Respiratory
alkalosis may be produced iatrogenically during excessive mechanical ventilation. Other causes include psychiatric (anxiety and
hysteria); CNS (stroke, subarachnoid haemorrhage, meningitis); pulmonary, such as pulmonary embolism; high altitude areas, where
the low atmospheric pressure of oxygen stimulates increased ventilation; fever, which stimulates the respiratory centre in the brainstem;
and pregnancy.

Mixed acid-base disorder (choice E) would result from cases such as salicylate poisoning. Salicylates directly stimulate the respiratory
center to cause hyperventilation (respiratory alkalosis) which is dose-dependent. Metabolic acidosis is the most serious acid-base
disorder and is due to increased production of endogenous acids rather than the salicylate itself. The combination of metabolic acidosis
and respiratory alkalosis can be a difficult situation to diagnose from the blood gases.

A 25 year old arrives comatose in an ambulance to the local emergency department. No history is available.
Vital signs are obtained:

pulse 110/min
regular temp 36.5C
respirations 7/min
blood pressure 118/76

Physical exam is otherwise unremarkable. The chest x-ray is negative.


In this clinical setting, which one of the following sets of arterial blood gases is to be expected?
1)
2)
3)
4)
5)

ph 7.50 pCO2 20, pO2 105


ph 7.39 pCO2 40, pO2 95
ph 7.20 pCO2 35, pO2 70
ph 7.19 pCO2 65, pO2 55
ph 7.18 pCO2 33, pO2 35

The correct answer is : C

Metabolic acidosis is a condition that occurs when the body produces too much acid or when the kidneys are not removing enough acid
from the body. If unchecked, metabolic acidosis leads to acidemia, i.e., blood pH is low (less than 7.35) due to increased production of
hydrogen by the body or the inability of the body to form bicarbonate (HCO3-) in the kidney. Its causes are diverse, and its
consequences can be serious, including coma and death

Symptoms are not specific, and diagnosis can be difficult unless the patient presents with clear indications for arterial blood gas
sampling. Symptoms may include chest pain, palpitations, headache, altered mental status such as severe anxiety due to hypoxia,
decreased visual acuity, nausea, vomiting, abdominal pain, altered appetite (either loss of or increased) and weight loss (longer term),
muscle weakness and bone pains. Those in metabolic acidosis may exhibit deep, rapid breathing called Kussmaul respirations which is
classically associated with diabetic ketoacidosis. Rapid deep breaths increase the amount of carbon dioxide exhaled, thus lowering the
serum carbon dioxide levels, resulting in some degree of compensation. Over compensation via respiratory alkalosis to form an
alkalemia does not occur.

Arterial blood gas sampling is essential for the diagnosis. The pH is low (under 7.35) and the bicarbonate levels are decreased (<24
mmol/l). Due to respiratory compensation (hyperventilation), carbon dioxide is decreased and conversely oxygen is increased.

To distinguish between the main types of metabolic acidosis, a clinical tool called the anion gap is considered very useful. It is calculated
by subtracting the chloride and bicarbonate levels from the sodium.

Anion gap = ( [Na+] ) - ( [Cl-]+[HCO3-] )

As sodium is the main extracellular cation, and chloride and bicarbonate are the main anions, the result should reflect the remaining
anions. Normally, this concentration is about 8-16 mmol/l (124). An elevated anion gap (i.e. > 16 mmol/l) can indicate particular types
of metabolic acidosis, particularly certain poisons, lactate acidosis and ketoacidosis.

ph 7.50 pCO2 20, pO2 105 (choice A) is consistent with a patient in respiratory alkalosis. Patients will exhibit symptoms of hypocalcemia
such as tetany with no fall in total serum calcium levels.

ph 7.39 pCO2 40, pO2 95 (choice B) is consistent with a a normal arterial blood gas.

pH 7.19 pCO2 65, pO2 55 (choice D) is consistent with a patient in respiratory acidosis with decreased respiration (hypoventilation) and
increased PaCO2 (hypercapnia). Respiratory acidosis can result from problems with the lung (COPD or ARDS) or problems outside the
lung that impairs respiration as in the case of neuromuscular disorders (e.g. Myasthenia gravis).

ph 7.18 pCO2 53, pO2 35 (choice E) is consistent with a mixed acid-base disorder such as salicylate poisoning. Salicylates directly
stimulate the respiratory center to cause hyperventilation (respiratory alkalosis) which is dose-dependent. Metabolic acidosis is the most
serious acid-base disorder and is due to increased production of endogenous acids rather than the salicylate itself. The combination of
metabolic acidosis and respiratory alkalosis can be a difficult situation to diagnose from the blood gases alone.

A 62 year old diabetic man in early stages of diabetic nephropathy presented with complaints of increasing fatigue and listlessness. He
did not have any overt symptoms of renal failure. The doctor advised routine blood test with blood gas analysis. Complete blood count
was normal; blood gas and serum electrolyte analysis gave the following results.

pH: 7.30 (7.36-7.44) Na+: 135 (136-146 mmol/L)


pCO2 :34 K+: 5.2 (3.5-5.0 mmol/L)
Cl- :110 (95-106 mmol/L) HCO3- :16 (22-29 mmol/L)
pO2: 92 Urine pH: 5.0

Which of the following conditions do not hold true for this patients acid base analysis?
1) Non-gap metabolic acidosis
2) Respiratory compensation is absent
3) Secondary disorder is absent

4) Renal tubule acidosis may be present


5) Hyperkalemia secondary to impaired secretion

The correct answer is : B.

To calculate whether the degree of compensation is adequate in case of metabolic acidosis, the following formula is used
pCO2~1.5(HCO3-)+8, in this case 1.5(16)+8=34, thus respiratory compensation is adequate.

If the serum pH < 7.35 and pCO2 < 40 metabolic acidosis.

If the pH < 7.35 and pCO2 > 40 respiratory acidosis.

Similarly, pH > 7.45 and pCO2 < 40 respiratory alkalosis.

And pH > 7.45 and pCO2 > 40 metabolic acidosis.

To calculate anion gap, Na+ - Cl- - HCO3- > 12, which in this case is 135-110-16=9. Thus there is no anion gap. A non-gap metabolic
acidosis is present (choice A).

Since anion gap is not elevated and respiratory compensation is appropriate, a secondary disorder is unlikely to be present (choice C).

The low urine pH along with non-gap metabolic acidosis is highly suggestive of a renal tubule acidosis (RTA) (choice D). Although the

patient history does not mention an etiology of type I (distal) or type II (proximal) RTA, however diabetics are at a risk of developing type
IV RTA. Hypoaldoteronsim leads to impaired acid secretion in the distal tubule resulting in type IV RTA. Also,in diabetic patients, type IV
RTA is most often due to the impaired conversion of prorenin to renin.

The hyperkalemia (choice E) may be due to impaired excretion of potassium in the distal tubule, as a consequence of low aldosterone
levels.
A 56 year female is brought into the emergency department by ambulance for acute exacerbation of her asthma. She has been
hospitalized in the past for COPD exacerbation and once was intubated. On exam her temperature is 37.8 degrees, BP 110/60 HR
110/bpm and she is breathing at 28 breaths/minute. She is in moderate distress and unable to talk in full sentences. Diffuse expiratory
wheezes are heard throughout the lungs. An arterial blood gas is taken and decision is made to intubate and place her on mechanical
ventilation.
All the following would contribute to her respiratory alkalosis except?
1)
2)
3)
4)
5)

assited ventilation
salicylic poisoning
COPD
pulmonary embolus
pulmonary edema

The correct answer is : C

Chronic respiratory acidosis may be secondary to many disorders, including COPD. Hypoventilation in COPD involves multiple
mechanisms, including decreased responsiveness to hypoxia and hypercapnia, increased ventilation-perfusion mismatch leading to
increased dead space ventilation, and decreased diaphragm function secondary to fatigue and hyperinflation

Respiratory alkalosis would result from increased respiration (hyperventilation) that elevates the blood pH. Respiratory alkalosis may be
produced iatrogenically during excessive mechanical ventilation. Other causes include psychiatric (anxiety and hysteria); CNS (stroke,
subarachnoid haemorrhage, meningitis); pulmonary, such as pulmonary embolism; high altitude areas, where the low atmospheric

pressure of oxygen stimulates increased ventilation; fever, which stimulates the respiratory centre in the brainstem; and pregnancy.

There are two types of respiratory alkalosis: chronic and acute. Acute respiratory alkalosis occurs rapidly. During acute respiratory
alkalosis, the person may lose consciousness where the rate of ventilation will resume to normal. Chronic respiratory alkalosis is a more
long-standing condition. For every 10 mM drop in pCO2 in blood, there is a corresponding 5 mM of bicarbonate ion drop. The drop of 5
mM of bicarbonate ion is a compensation effect which reduces the alkalosis effect of the drop in pCO2 in blood. This is termed
metabolic compensation.

assisted ventillation (choice A) is a common iatrogenically induced respiratory alkalosis from increased minute ventilation.

salicylic poisoning (choice B) would result in dose dependent respiratory alkalosis. Salicylates directly stimulate the respiratory center to
cause hyperventilation.

pulmonary embolus (choice D) symptoms include tachypnea, tachycardia, chest pain on inspiration, and palpitations. Clinical signs
include low blood oxygen saturation and cyanosis, rapid breathing, and a rapid heart rate. The tachypnea can become pronounced and
patients can expel a large amount of CO2, resulting in respiratory alkalosis.

pulmonary edema (all causes) (choice E) would result in a metabolic alkalosis with respiratory compensation.

A 65 year man was found in a state of clouded consciousness. At the hospital, his Glasgow Coma Scale score was 9. The serum anion
gap was 1 mmol/L, and the osmolar gap 12.8 mOsmol/kg. At the time of hospital admission, the patients blood pressure was 91/64 mm
Hg, heart rate 111 beats/min, respiratory rate 22 and temperature 35.6C. His urine output over 6 hours was 2.4 L and dilute in
appearance. The patient had a history of suicidal ideation, bipolar affective disorder and hypertension and was being treated for the

same. Blood gas analysis provided the following data (normal values are given in the bracket).

Sodium, mmol/L (135145) 143


Potassium, mmol/L (3.25.0) 3.6
Chloride, mmol/L (100110) 109
Bicarbonate, mmol/L (2329) 36
Creatinine, mol/L (< 99) 95
Calcium, mmol/L (2.202.62) 2.68
Albumin, g/L (3545) 40
Plasma glucose, mmol/L (3.87) 4.0
pH (7.357.45) 7.38

What is the most likely cause of this patients low anion gap?
1)
2)
3)
4)
5)

Metabolic alkalosis
Ethylene glycol
Lithium overdose
Hypercalecemia
Metabolic acidosis

The correct answer is : C.

The anion gap is a key diagnostic clue when approaching the differential diagnosis of toxidromes and metabolic acidosis. Since the
patient is on medication for bipolar disorder and suicidal ideation, it should be checked whether he is on lithium therapy. Lithium is a
positive charged ion with valence of 1 and electrochemical properties similar to those of sodium and potassium. Large quantities of
positive cations in the plasma are balanced by the anions bicarbonate and chloride and can cause a negative anion gap because
sodium, but not lithium, is included in the calculation.

Lithium is highly effective in the treatment of bipolar disease, however, because of the drugs narrow therapeutic window, toxic effects
are common. The most common manifestation of toxicity is altered mental status. The most common manifestation of toxicity is altered
mental status. Our patient also had a high urine output of 2.4 L in 6 hours, which suggests polyuria due to a concentrating defect.
Depending on the length of lithium use, toxic effects can occur with levels as low as 2.5 mmol/L.

The osmolar gap is another useful diagnostic clue in cases of overdose. In this patient, the osmolar gap was elevated at 36 mOsmol/kg.
Elevated osmolar gap can be caused due to isopropanolol, N-propanolol, propylene glycol, ethylene glycol, methanol, formaldehyde,
paraldehyde, mannitol, diethyl ether ingestion and lithium overdose.

The high level of lithium with its anion, bicarbonate (from lithium carbonate), contributed to an increase in plasma osmolality. This is
further supported by the patients elevated bicarbonate level of 36 on presentation. The expected increase in osmolar gap would be 2
times the lithium level, or about 28 mOsmol/kg, which is similar to the level in our patient. The combination of an elevated osmolar gap
and decreased anion gap without metabolic acidosis should be considered highly suggestive of severe lithium intoxication after
exclusion of severe hyperproteinemia, hyperlipidemia and mannitol ingestion.

Since, the patient is probably on long term lithium therapy, it may be the most obvious reason. A serum and urine toxicology screen
would be positive for lithium and confirm the diagnosis. Other causes can be excluded if the results of the toxicology screen were
negative for alcohols, serum ketones and lactate and the patient had a normal lipid profile.

Low anion gap can be caused due to hypercalcemia, hypermagnesemia, hyperkalemia, cationic immunoglobulins (as in plasma cell
dyscrasias), bromide intoxication, nitrates or lithium.

Ethylene glycol (choice B) intake would result in increase in osmolar gap but not low anion gap. Since, blood gas analysis shows the
values of most anions as normal, hypercalcemia (choice D) can be ruled out.
Since pH is within the normal range metabolic alkalosis (choice A) or acidosis (choice E) can also be ruled out.
You are evaluating a 5 year old male who was recently adopted from an orphanage in China. You review his medical records and see
that he is reportedly well, has had no surgeries, and is taking no medications. Previous laboratory testing shows a normal blood lead
level and a negative HIV test. The boy has never received immunizations for varicella or MMR, however. The child's height is 120 cm
(97th percentile), weight 22 kg (85th percentile), and blood pressure is 88/60. On examination, you note a well-appearing child whose
lungs are clear to auscultation. You obtain a chest x-ray, which is normal, and then place a tuberculin purified protein derivative (PPD)

in the patient's forearm.


Which of the following would indicate a positive PPD in this patient?
1) >1 mm
2) >5 mm
3) >10 mm
4) >15 mm
5) >20 mm
The correct answer is : C.

Interpretation of a patient's PPD depends greatly on the pre-test likelihood that the patient has tuberculosis, and on their likelihood of
mounting an adequate immune response.

Thus, for patients who are the highest risk - such as those with known HIV, previous contact with a known case, or who have an
abnormal chest x-ray - a cutoff of 5 mm should be used.

For patients with moderate risk (including birth or residence in a country with high rates of tuberculosis, residence in a prison or longterm care facility, employment in healthcare professions, or chronic medical conditions like diabetes or renal failure), a positive PPD is
10 mm or greater. Also included in this category are children younger than 4 years old, as they have a higher likelihood of
demonstrating anergy to the PPD.

When no risk factors can be elicited by history, a cutoff of 15 mm is used.

Choices A, B, D, and E are inappropriate cutoffs for a positive PPD in this moderate risk patient who was born in a nation with high rates
of tuberculosis and who has resided in a high risk environment (an orphanage).

A 29 year old man with a history of type 1 diabetes presented to the emergency with a 4-day history of nausea, vomiting, and flu-like

symptoms. He was single, unemployed, did not drink alcohol, has no previous psychiatric history, no family history of diabetes or other
medical conditions. Physical examination revealed a temperature of 36.4C (97.5F), heart rate of 106 beats per minute, supine blood
pressure of 131/85 mmHg, and sitting blood pressure of 122/80 mmHg. He appeared clinically dehydrated with dry oral mucosa, but
cardiovascular, respiratory, abdominal, and neurological examinations were otherwise normal. He admitted to not having eaten for
nearly 2-3 weeks prior to his presentation to the hospital and was subsequently diagnosed and treated for depression.
Metabolic acidosis was confirmed with the following lab tests:

pH: 7.3 HCO3-: 10 mEq/l anion gap: 29 mEq/l


Urine analysis confirmed heavy (4+) ketonuria
Amylase of 104 IU/l (normal range: <150 IU/l)
Lactate of 9.9 mg/dl (1.1 mmol/l) (normal range: 0.5 2.2 mmol/l).

However, blood glucose concentration was normal 105 mg/dl (normal 70-110 mg/dL). Blood salicylate, alcohol, and methanol levels
were all undetectable.
What is the correct diagnosis for this patient?
1)
2)
3)
4)
5)

Euglycemic ketoacidosis
Diabetic ketoacidosis
Lactic acidosis
Alcoholic ketoacidosis
Starvation acidosis

The correct answer is : A

A diagnosis of euglycemic ketoacidosis was made with the precipitating factor thought to be a viral gastroenteritis. He was treated with
0.9% saline rehydration, 10% dextrose, and intravenous insulin and his acidosis gradually improved. Vitamins B including thiamine and
vitamin C were also administered intravenously three times a day during the first 72 h of admission.

Diabetic ketoacidosis (DKA) (choice B) is a state of absolute or relative insulin deficiency and is defined by the American Diabetes
Associations (ADA) diagnostic criteria of hyperglycemia [blood glucose >250mg/dl (13.9mmol/l)], acidosis (arterial pH<7.3 and serum

bicarbonate <15mEq/l), and ketosis (moderate ketonuria or ketonemia).

This case highlights that even in the absence of hyperglycemia, acid/base status should be assessed in an ill patient with diabetes, and
in cases of euglycemic diabetic ketoacidosis, the diagnosis of depression should be considered as a cause for suppressed appetite and
anorexia. In clinical practice it must be remembered that patients with diabetes may be at risk of developing acidosis from conditions
that are also seen in patients without diabetes, and a distinction between non-diabetic euglycemic ketoacidosis and euglycemic DKA is
essential. The former includes in the differential diagnoses, such as prolonged starvation, excess alcohol consumption, salicylate
overdose, lactic acidosis, tricyclic antidepressant overdose, and renal tubular acidosis, while the latter is a complex pathophysiological
process specific to individuals with severe insulin deficiency. These conditions should not only be excluded in diabetic patients with
normal or mildly elevated glucose concentrations, but should also be considered in situations of overt hyperglycemia where it may be
suspected that the diabetic condition may not be the sole contributor to the acidotic state.

Starvation ketoacidosis (choice E) can be differentiated from euglycemic DKA by clinical assessment (one would usually expect the
presence of an intercurrent illness to act as a precipitant for euglycemic DKA, with the starvation possibly occurring as a result of the
intercurrent illness rather than as a primary event) and measurement of the serum bicarbonate concentration, which in starvation ketosis
is usually not lower than 18 mEq/l1. In practice there may be a considerable degree of overlap between starvation ketoacidosis and
euglycemic DKA, as the relative normoglycemia in euglycemic DKA occurs as a result of prolonged fasting. The correct diagnosis of
euglycemic DKA is also necessary to tailor therapy accordingly. The initial management should be, as in any case of ketoacidosis, to
correct fluid/electrolyte abnormalities and re-establish carbohydrate metabolism. Increased glucose administration using higher
percentages of dextrose (10 or 20%) are required to facilitate the concomitant administration of the relatively large amounts of insulin
that are needed to correct the severe acidosis in these patients. Acidosis should improve with normalization of serum bicarbonate
without the need for intravenous bicarbonate administration. A decrease in caloric intake is frequently associated with the development
of DKA, usually due to nausea or vomiting caused by a precipitating illness or by worsening ketoacidosis itself.

Causes of Euglycemic Ketoacidosis include starvation, vomiting (ketoacidosis, gastroparesis), intentional dieting, depression,
anhedonia, pregnancy, glycogen storage disorders, cirrhosis, as a result of H+ excess, salicylate overdose, methanol ingestion, lactic
acidosis, pancreatitis, sepsis, and renal tubular acidosis.

Lactic acidosis (choice C) is identified by a state of acidosis and an elevated plasma lactate concentration. It is one type of anion gap
metabolic acidosis and may result from numerous conditions. The normal blood lactate concentration in unstressed patients is 1-0.5
mmol/L. Patients with critical illness can be considered to have normal lactate concentrations of less than 2 mmol/L. Hyperlactatemia is
defined as a mild-to-moderate persistent increase in blood lactate concentration (2-5 mmol/L) without metabolic acidosis, whereas lactic

acidosis is characterized by persistently increased blood lactate levels (usually >4-5 mmol/L) in association with metabolic acidosis. The
serum lactate concentration in this patient is 1.1 mmol/l, thus lactate acidosis is absent.

Alcoholic ketoacidosis (choice D) is an acute metabolic acidosis seen in those with a recent history of binge drinking and little or no
nutritional intake. Previously, diabetes mellitus was thought to be the most common cause of a severe ketoacidosis. Alcoholic
ketoacidosis is characterized by high serum ketone levels and an elevated anion gap. A concomitant metabolic alkalosis is also
common, resulting from vomiting and volume depletion. Although AKA most commonly occurs in adults with alcoholism, alcoholic
ketoacidosis has been reported in less-experienced drinkers of all ages.

A 15 year old girl develops fever, sore throat, and malaise over the past 8 days. On examination, you find a somewhat ill apearing
female with moderate facial edema. There is erythema of the posterior oropharynx with thick yellow exudates, and numerous 1-2 cm
tender lymph nodes in the cervical chains. A CBC shows a WBC 9,500/mcL with 25% polymorphonuclear lymphocytes,
67% lymphocytes, and 8% monocytes; hemoglobin of 9.5 g/dL; and platelet count 103,000/mcL. Several atypical lymphocytes are seen
on a peripheral smear.
Which of the following is the most likely associated finding in this patient?
1)
2)
3)
4)
5)

Conjunctivitis
Glomerulonephritis
Neutropenia
Peritonsillar abscess
Splenomegaly

The correct answer is : E.

This patient has infectious mononucleosis, caused by Epstein-Barr virus (EBV) infection. Key features include fever and malaise;
exudative pharyngitis; and atypical lymphocytosis. Splenomegaly commonly develops in patients with EBV infection, and may often be
a clue to the diagnosis. Another commonly tested clue to a diagnosis of EBV mononucleosis is the appearance of a fullbody maculopapular rash when patients receive treatment with ampicillin or amoxicillin.

Conjunctivitis (choice A) can occur as a postinfectious complication commonly called Reiter's syndrome, consisting of the triad of
arthritis, urethritis, and conjunctivitis. Commonly implicated organisms include Chlamydia trachomatis and Campylobacter.

Acute glomerulonephritis (choice B) may complicate untreated group A, beta-hemolytic Streptococcus pyogenesinfections.

Neutropenia (choice C) is a common complication of a variety of self-limited viral infections, caused by viral suppression of the bone
marrow. It is usually self-limited and does not cause any clinically significant problems.

Peritonsillar abscess (choice D) is a frequent complication of streptococcal pharyngitis.

A 67 year old woman with type II diabetes, congestive heart failure, and beta-thalassemia is brought to the emergency from a nursing
home with a three day history of pneumonia. She has been on total parental nutrition ever since undergoing a hemicolectomy a year
ago. On hospital day three, the patient develops severe sepsis and is sent to the MICU. An arterial blood gas is obtained which shows a
PaCO2 of 65 mmHg., PaO2 of 45 mmHg and a blood pH of 7.23 and it is determined that she is in severe metabolic acidosis.
All of the following are causes of lactic acidosis type A, except?
1)
2)
3)
4)
5)

Septic shock
Severe anemia
Metformin
Cyanide poisoning
Respiratory failure

The correct answer is : C.

Lactic acidosis is a physiological condition characterized by low pH in body tissues and blood accompanied by the buildup of lactate,

and is considered a distinct form of high anion gap metabolic acidosis. The condition typically occurs when tissues receive too little
oxygen (hypoxia or hypoperfusion) and cells are forced to metabolize glucose anaerobically. Therefore, excess pyruvate produced is
converted into lactate and released into the bloodstream. Lactic acidosis is characterized by lactate levels >5 mmol/L and serum pH
<7.35.

Signs of lactic acidosis are deep and rapid breathing (Kussmaul breathing), vomiting, and abdominal painsymptoms that may easily
be mistaken for other problems. Lactic acidosis may be caused by diabetic ketoacidosis or kidney and liver disease. Heavy metals
(arsenic and cyanide) and medications such as metformin and anti-HIV drugs (antiretrovirals) can also cause lactic acidosis by
mitochondrial toxicity.

The Cohen-Woods classification categorizes causes of lactic acidosis as follows:

Type A: Decreased perfusion or oxygenation

Overproduction: Circulatory, pulmonary, or hemoglobin transfer disorders are commonly responsible.

Underutilization: Liver disease, gluconeogenesis inhibition, thiamine deficiency, and uncoupled oxidative phosphorylation

Type B: Evidence of poor tissue perfusion or oxygenation is absent

B1: Systemic disease such as renal and hepatic failure, diabetes and malignancy.

B2: Medication or intoxication (biguanides, alcohols, iron, isoniazid, zidovudine and salicylates)

B3: Inborn error of metabolism

Septic shock (choice A) decreases tissue perfusion and oxygen delivery as a result of infection and sepsis, though the microbe maybe
localized to a particular site.

Severe anemia (choice B) decreases tissue perfusion and oxygen delivery as a result of the decreased oxygen carrying capability of the
blood. There may be signs of a hyperdynamic circulation: a fast heart rate (tachycardia), flow murmurs, and cardiac enlargement as the
body tries to compensate with increased cardiac output.

Thiamine deficiency (choice D) leads to impaired aerobic respiration and is often prevalent in patients with impaired nutrition (chronic
alcoholism, malaborption syndromes, HIV-AIDS, parenteral glucose or hyperalimentation without adequate B-vitamin supplementation)

Respiratory failure (choice E) decreases tissue perfusion and oxygen delivery due to inadequate gas exchange by the respiratory
system. Therefore, arterial oxygen and/or carbon dioxide levels cannot be maintained within their normal ranges. A drop in blood
oxygenation is known as hypoxemia; a rise in arterial carbon dioxide levels is called hypercapnia. The normal reference values are:
oxygen PaO2 greater than 60 mmHg , and carbon dioxide PaCO2 less than 45 mmHg.

A 57 year-old obese man who has been recovering from a knee sprain develops sudden shortness of breath and left-sided pleuritic
chest pain with inspiration. On exam, HR is 137, RR is 24, Temp is 37 C, the pulmonic portion of the second heart sound is
accentuated, 3 cm of JVD is appreciated, and the right leg is slightly swollen and tender. Trachea is midline and lung sounds are equal
bilaterally without wheezes, rales, or rhonchi. The remainder of physical exam is unremarkable.

Which of the following would correctly describe her acid-base status?


1)
2)
3)
4)
5)

metabolic acidosis
metabolic alkalosis
respiratory acidosis
respiratory alkalosis
mixed acid-base disorder

The correct answer is : D

Pulmonary embolism (PE) is a blockage of the main artery of the lung or one of its branches by a substance that has travelled from
elsewhere in the body through the bloodstream (embolism). Usually this is due to embolism of a thrombus (blood clot) from the deep
veins in the legs, a process termed venous thromboembolism. A small proportion is due to the embolization of air, fat or amniotic fluid.
The obstruction of the blood flow through the lungs and the resultant pressure on the right ventricle of the heart leads to the symptoms
and signs of PE. The risk of PE is increased in various situations, such as cancer (hypercoaguable states), prolonged bed rest, and
recent trauma.

Symptoms of pulmonary embolism include tachypnea, tachycardia, chest pain on inspiration, and palpitations. Clinical signs include low
blood oxygen saturation and cyanosis, rapid breathing, and a rapid heart rate. The tachypnea can become pronounced and patients can
expel a large amount of CO2, resulting in respiratory alkalosis.

Metabolic acidosis (choice A) would arise in situations such as diarrhea (non gap acidosis) or the MUDPILES high anion gap acidosis
(methanol intoxication, uremia, diabetic ketoacidosis, para-aldehyde, isoniazid/iron, lactic acid, ethyleneglycol, salicylates).

Metabolic alkalosis (Choice B) would result from decreased hydrogen ion concentration (vomitting or excess aldosterone), leading to
increased bicarbonate, or alternatively a direct result of increased bicarbonate concentrations (hypokalemia or use of alkalotic agents in
peptic ulcer disease). Generally, metabolic alkalosis can be divided into two categories, depending upon urine chloride levels: chloride-

responsive (<10 mEq/L) or Chloride-resistant (>20 mEq/L) metabolic alkalosis.

Respiratory acidosis (choice C) would result from decreased respiration (hypoventilation) and increased PaCO2 (hypercapnia).
Respiratory acidosis can result from problems with the lung (COPD or ARDS) or problems outside the lung that impairs respiration as in
the case of neuromuscular disorders (e.g. Myasthenia gravis).

Mixed acid-base disorder (choice E) would result from cases such as salicylate poisoning. Salicylates directly stimulate the respiratory
center to cause hyperventilation (respiratory alkalosis) which is dose-dependent. Metabolic acidosis is the most serious acid-base
disorder and is due to increased production of endogenous acids rather than the salicylate itself. The combination of metabolic acidosis
and respiratory alkalosis can be a difficult situation to diagnose from the blood gases.

You are consulted on a 67-year-old woman with severe COPD who was intubated on the medicine floor 30 minutes earlier because of
respiratory distress. Her temperature is 37.0 C (98.6 F), blood pressure is 90/60 mm Hg, pulse is 133/min and regular, and respiratory
rate is 24/min. Her ABG reveals the following: pH 7.21, PaCO2 80 mm Hg, PaO2 69 mm Hg, and her oxygen saturation is 91%. Her
current ventilator settings are assist control (AC), respiratory rate (RR) of 12/min, tidal volume (TV) of 500 ml, fraction of inspired oxygen
(FiO2) is 50%, positive end expiratory pressures (PEEP) of 5 mm/Hg.
Which of the following would correctly describe her acid-base status?
1)
2)
3)
4)
5)

metabolic acidosis
metabolic alkalosis
respiratory acidosis
respiratory alkalosis
mixed acid-base disorder

The correct answer is : C

Respiratory acidosis is a medical condition in which decreased respiration (hypoventilation) causes increased blood carbon dioxide and

decreased pH.
Carbon dioxide is produced constantly as the body burns energy, and this CO2 will accumulate rapidly if the lungs do not adequately
dispel it through alveolar ventilation. Alveolar hypoventilation thus leads to an increased PaCO2 (hypercapnia).

In chronic respiratory acidosis, the PaCO2 is elevated above the upper limit of the reference range, with a normal blood pH (7.35 to
7.45) or near-normal pH secondary to renal compensation and an elevated serum bicarbonate (HCO3 >30 mm Hg).

Chronic respiratory acidosis may be secondary to many disorders, including COPD. Hypoventilation in COPD involves multiple
mechanisms, including decreased responsiveness to hypoxia and hypercapnia, increased ventilation-perfusion mismatch leading to
increased dead space ventilation, and decreased diaphragm function secondary to fatigue and hyperinflation.

Chronic respiratory acidosis also may be secondary to obesity hypoventilation syndrome (i.e., Pickwickian syndrome), neuromuscular
disorders such as amyotrophic lateral sclerosis, and severe restrictive ventilatory defects as observed in interstitial fibrosis and thoracic
deformities.

Metabolic acidosis (choice A) would arise in situations such as diarrhea (non gap acidosis) or the MUDPILES high anion gap acidosis
(methanol intoxication, uremia, diabetic ketoacidosis, para-aldehyde, isoniazid/iron, lactic acid, ethyleneglycol, salicylates).

Metabolic alkalosis (Choice B) would result from decreased hydrogen ion concentration (vomitting or excess aldosterone), leading to
increased bicarbonate, or alternatively a direct result of increased bicarbonate concentrations (hypokalemia or use of alkalotic agents in
peptic ulcer disease). Generally, metabolic alkalosis can be divided into two categories, depending upon urine chloride levels: chlorideresponsive (<10 mEq/L) or Chloride-resistant (>20 mEq/L) metabolic alkalosis.

Respiratory alkalosis (choice D) would result from increased respiration (hyperventilation) that elevates the blood pH. Respiratory
alkalosis may be produced iatrogenically during excessive mechanical ventilation. Other causes include psychiatric (anxiety and
hysteria); CNS (stroke, subarachnoid haemorrhage, meningitis); pulmonary, such as pulmonary embolism; high altitude areas, where

the low atmospheric pressure of oxygen stimulates increased ventilation; fever, which stimulates the respiratory centre in the brainstem;
and pregnancy.

Mixed acid-base disorder (choice E) would result from cases such as salicylate poisoning. Salicylates directly stimulate the respiratory
center to cause hyperventilation (respiratory alkalosis) which is dose-dependent. Metabolic acidosis is the most serious acid-base
disorder and is due to increased production of endogenous acids rather than the salicylate itself. The combination of metabolic acidosis
and respiratory alkalosis can be a difficult situation to diagnose from the blood gases.

A 67 year old homeless man is brought to the emergency room. He is diabetic and has a history of admissions for delirium tremens. He
is minimally compliant with his dialysis appointments. On physical exam, the patient is unarousable even with sternal rubs and the rest
of the exam is unremarkable. His vitals are temperature of 37.2 degrees, BP 115/70, HR 95 and RR of 22.
A stat metabolic panel reveals:

Na: 137
K: 3.6
Cl: 105
HCO3: 20
BUN: 60
Cr: 3.4
Glucose: 342

An arterial blood gas is obtained which shows a PaCO2 of 30 mmHg., PaO2 of 45 mmHg and a blood pH of 7.23 and it is determined
that he is in severe metabolic acidosis.
Which of the following is the cause of the patients metabolic acidosis?
1)
2)
3)
4)
5)

diabetic ketoacidosis
renal tubular acidosis
acute tubular necrosis
ethylene glycol intoxication
lactic acidosis

The correct answer is : B

To distinguish between the main types of metabolic acidosis, a clinical tool called the anion gap is considered very useful. It is calculated
by subtracting the chloride and bicarbonate levels from the sodium.

Anion gap = ( [Na+] ) - ( [Cl-]+[HCO3-] )

As sodium is the main extracellular cation, and chloride and bicarbonate are the main anions, the result should reflect the remaining
anions. Normally, this concentration is about 8-16 mmol/l (124). An elevated anion gap (i.e. > 16 mmol/l) can indicate particular types
of metabolic acidosis, particularly certain poisons, lactate acidosis and ketoacidosis.
Caluclating this patients anion gap reveals a normal anion gap, which shortens the differential diagnosis considerably.

The most common causes of normal anion gap acidosis are GI or renal bicarbonate loss and impaired renal acid excretion. Normal
anion gap metabolic acidosis is also called hyperchloremic acidosis, because instead of reabsorbing HCO3 with Na, the kidney
reabsorbs Cl. Many GI secretions are rich in bicarbonate (eg, biliary, pancreatic, and intestinal fluids); loss from diarrhea, tube
drainage, or fistulas can cause acidosis. In ureterosigmoidostomy (insertion of ureters into the sigmoid colon after obstruction or
cystectomy), the colon secretes and loses bicarbonate in exchange for urinary Cl and absorbs urinary ammonium, which dissociates
into NH3+ and H+.

A useful mnemonic that summarizes the causes of normal anion gap acidosis is DURHAM

Diarrhea (HCO3- and water is lost).

Ureteral diversion: Urine from the ureter may be diverted to the sigmoid colon due to disease (uretero-colonic fistula) or after
bladder surgery. In such an event urinary Cl- is absorbed by the colonic mucosa in exchange for HCO3-, thus increases the
gastrointestinal loss of HCO3-.

Renal tubular acidosis: dysfunctional renal tubular cells causes an inappropriate wastage of HCO3- and retention of Cl-.

Hyperalimentation

Acetazolamide

Miscellaneous conditions: They include pancreatic fistula, cholestyramine, and calcium chloride (CaCl) ingestion, all of which can
increase the gastrointestinal wastage of HCO3-.

Diabetic ketoacidosis (choice A) would lead to a high anion gap acidosis.

Acute tubular necrosis (chocie C) is caused by acute kidney injury due to low blood volume, exposure to toxins, and prostate
enlargement. Depending on its severity, AKI may lead to a number of complications, including high anion gap metabolic acidosis from
uremia, high potassium levels, changes in body fluid balance, and effects to other organ systems. Management includes supportive
care, such as renal replacement therapy, as well as treatment of the underlying disorder.

Ethylene glycol intoxication (choice D) would lead to a high anion gap acidosis.

Lactic acidosis (Choice E) would lead to a high anion gap acidosis.

A 58 year old man is brought to the emergency department from after being found unresponsive. He has a long history of admissions
related to uncontrolled diabetic mellitus, renal failure and alcohol related seizures. The patient was dialyzed last week and a finger stick
reveals a blood glucose of 140. Upon further inspection, the patient is nonarousable with moist mucous. He is breathing deeply and
rapidly and the decision is made to intubate the patient. An arterial blood gas is obtained which shows a PaCO2 of 65 mmHg., PaO2 of
45 mmHg and a blood pH of 7.23 and it is determined that he is in severe metabolic acidosis.
Which of the following metabolites would be elevated in the patients blood?
1)
2)
3)
4)
5)

Formic acid
Lactic acid
Acetoacetic acid and beta hydroxybutyrate
Glycolic acid
Phosphoric acid and sulphuric acid

The correct answer is : D.

Ethanol is metabolized by the enzyme alcohol dehydrogenase (ADH) pathway, which is located in the liver and gastric mucosa, and by
the cytochrome P-450 mixed function oxidase (MFO) system in the liver. As with ethyl alcohol and methanol, ethylene glycol is
metabolized by alcohol dehydrogenase to form glycoaldehyde. Through interaction with aldehyde dehydrogenase, ethylene glycol is
then metabolized to glycolic acid. Glycolic acid and its toxic byproducts first affect the central nervous system, then the heart, and finally
the kidneys. A profound acidosis often ensues and is attributable to the glycolic acid in circulation. The patient may develop
hyperventilation and Kussmaul respirations that results from high anion gap acidemia.

Formic acid (choice A) results from methanol poisoning and can cause permanent blindness by destruction of the optic nerve. The initial
symptoms of methanol intoxication include central nervous system depression, headache, dizziness, nausea, lack of coordination,
confusion, and with sufficiently large doses, unconsciousness and death. Methanol poisoning can be treated with the antidotes ethanol

or fomepizole.

Lactic acid (choice B) results from lactic acidosis and is considered a distinct form of high anion gap metabolic acidosis. The condition
typically occurs when tissues receive too little oxygen (hypoxia or hypoperfusion) and cells are forced to metabolize glucose
anaerobically. Therefore, excess pyruvate produced is converted into lactate and released into the bloodstream. Lactic acidosis is
characterized by lactate levels >5 mmol/L and serum pH <7.35.

Acetoacetic acid and beta hydroxybutyrate (choice C) results from ketoacidosis, which is caused by high concentrations of ketone
bodies formed by the breakdown of fatty acids and the deamination of amino acids. In diabetic patients, ketoacidosis is usually
accompanied by insulin deficiency, hyperglycemia, and dehydration. Particularly in type 1 diabetics the lack of insulin in the bloodstream
prevents glucose absorption and can cause unchecked ketone body production (through fatty acid metabolism) potentially leading to
dangerous glucose and ketone levels in the blood. Hyperglycemia results in glucose overloading the kidneys and spilling into the urine.
Dehydration results following the osmotic movement of water into urine, exacerbating the acidosis.

Phosphoric acid and sulphuric acid (choice E) results from chronic renal failure. If the renal damage affects both glomeruli and tubules,
the acidosis is a high-anion gap acidosis. It is due to failure of adequate excretion of various acid anions due to the greatly reduced
number of functioning nephrons. If the renal damage predominantly affects the tubules with minimal glomerular damage, a different type
of acidosis may occur. This is called Renal Tubular Acidosis (RTA) and this is a normal anion gap or hyperchloraemic type of acidosis.
The GFR may be normal or only minimally affected.

A 24 year old woman with no significant medical history comes to clinic for routine annual checkup. She has been under some stress at
work and her only complaint is new onset fatigue in the last month that is made worse with exertion and improved by periods of rest. Her
physical examination and routine lab testing is within normal limits. Her only imaging is a CT of the chest taken several months ago that
shows a large mass in the anterior mediastinum.
Which of the following would correctly describe her acid-base status?
1)
2)
3)
4)

Low pH, Low HCO3, High Base Deficit, Low pCO2


Low pH, High HCO3, High Base Excess, High pCO2
Neutal pH, Low HCO3, High Base Deficit, Low pCO2
High pH, High HCO3, High Base Excess, high pCO2

5) High pH, Low HCO3, High Base Excess, Low pCO2

The correct answer is : B.

Base excess is defined as the amount of strong acid that must be added to each liter of fully oxygenated blood to return the pH to 7.40
at a temperature of 37C and a pCO2 of 40 mmHg. Comparison of the base excess with the reference range assists in determining
whether an acid/base disturbance is caused by a respiratory, metabolic, or mixed metabolic/respiratory problem. While carbon dioxide
defines the respiratory component of acid-base balance, base excess defines the metabolic component.

The predominant base contributing to base excess is bicarbonate. Thus, a deviation of serum bicarbonate from the reference range is
ordinarily mirrored by a deviation in base excess. However, base excess is a more comprehensive measurement, encompassing all
metabolic contributions.

The patient has signs of mysthenia gravis. The hallmark of myasthenia gravis is fatiguability. Muscles become progressively weaker
during periods of activity and improve after periods of rest. Muscles that control eye and eyelid movement, facial expression, chewing,
talking, and swallowing are especially susceptible. The muscles that control breathing and neck and limb movements can also be
affected. Often the physical examination is within normal limits.

The onset of the disorder can be sudden. Often symptoms are intermittent. The diagnosis of myasthenia gravis may be delayed if the
symptoms are subtle or variable. In most cases, the first noticeable symptom is weakness of the eye muscles. In others, difficulty in
swallowing and slurred speech may be the first signs. The degree of muscle weakness involved in mysthenia gravis varies greatly
among patients, ranging from a localized form, limited to eye muscles (ocular myasthenia), to a severe or generalized form in which
many muscles - sometimes including those that control breathing - are affected. Symptoms, which vary in type and severity, may include
asymmetrical ptosis (a drooping of one or both eyelids), diplopia (double vision) due to weakness of the muscles that control eye
movements, unstable or waddling gait, weakness in arms, hands, fingers, legs, and neck, a change in facial expression, dysphagia
(difficulty in swallowing), shortness of breath and dysarthria (impaired speech, often nasal due to weakness of the velar muscles).

In myasthenic crisis a paralysis of the respiratory muscles occurs, necessitating assisted ventilation to sustain life. Subsequently,
patients develop respiratory acidosis due to inadeqaute aveolar ventilaltion. In patients whose respiratory muscles are already weak,
crises may be triggered by infection, fever, an adverse reaction to medication, or emotional stress. Since the heart muscle is only
regulated by the autonomic nervous system, it is generally unaffected by MG.

Low pH, Low HCO3, High Base Deficit, Low pCO2 (choice A) is consistent with metabolic acidosis such as diarrhea (non gap acidosis)
or the MUDPILES high anion gap acidosis (methanol intoxication, uremia, diabetic ketoacidosis, para-aldehyde, isoniazid/iron, lactic
acid, ethyleneglycol, salicylates).

Neutal pH, Low HCO3, High Base Deficit, Low pCO2 (choice C) is consistent with a mixed metabolic/respiratory problem such as
salicylate poisoning.

High pH, High HCO3, High Base Excess, high pCO2 (choice D) is consistent metabolic alkalosis such as excessive vomitting and
Cushings disease.

High pH, Low HCO3, High Base Excess, Low pCO2 (Choice E) is consistent with respiratory alkalosis as in hyperventilated states (e.g.
pulmonary embolism).

A 57-year-old male with COPD, home oxygen dependence, exercise tolerance of 20-30 yards, and daily chronic sputum production, is
on maintenance treatment with several types of metered dose inhalers and no other medications. He presents to your clinical with a 12
hour history of purulent sputum production, and 3-4 hours of worsening shortness of breath increased weakness. When you see him he
appears to be in moderate respiratory distress with peripheral cyanosis and mild confusion. You transfer him to the Emergency Room
where he develops worsening respiratory failure and requires mechanical ventilation. He is placed on a ventilator with tidal volume 800
ml, 100 percent FIO2, and rate of 12/min. Arterial blood gas is performed after 30 minutes on these ventilator settings.
Which of the following is the most likely acid-base disorder to be present at this time?
1) acute respiratory alkalosis and metabolic alkalosis

2)
3)
4)
5)

acute respiratory alkalosis and metabolic acidosis


chronic respiratory acidosis
acute respiratory alkalosis
no acid-base disorder is likely to develop

The correct answer is : A.

This man, by virtue of his severe COPD, will have developed chronic CO2 retention. Over the course of time his body will have
compensated for this respiratory acidosis by renal retention of bicarbonate. The effectiveness of this metabolic compensation for a
respiratory acidosis depends on the duration of the acidosis, and on normal renal handling of bicarbonate. In addition to the chronic
state he will have had an acute rise in pCO2 prior to initiation of ventilation.

The elevated CO2 level is dealt with acutely through the action of carbonic anhydrase inside red blood cells: CO2+ H2O <-> H2CO3 <->
H+ + HCO3-. The H+ generated in this process is then buffered by intracellular protein or phosphates. In acute respiratory acidosis there
is no time for renal compensation and so the [HCO3-] rises by only by around 1 mmol/L for every 10mmHg increase in pCO2 above its
reference value of 40mmHg.

The renal response to hypercapnic acidosis is at a maximum after 2-4 days, although it is already evident by 6-14 hours. At its peak,
there is a 4 mmol/L increase in [HCO3-] for every 10mmHg increase in pCO2 from the reference value of 40mmHg. So lets assume that
our patient in this case had a chronic arterial pCO2 of 60 mmHg. We could expect his bicarbonate to have risen by 8 mmol/L above a
normal of 25 mmol/L, giving us a bicarbonate level of 32 mmol/L at onset of his acute exacerbation. It will not have substantially
changed from this point.

The hypercapnia in COPD exacerbation and chronic COPD is due to a combination of increased deadspace, muscle weakness, and
some expiratory flow limitation with consequent gas trapping. Assuming wheeze is not a major component here, when we placed this
man on the mechanical ventilator and rested his respiratory muscles his CO2 production will have fallen and his CO2 excretion increased
very significantly. His minute ventilation may have doubled on these settings. The effect of this will be to acutely lower his arterial
pCO2 from its elevated level towards or even below normal. This will produce at best a situation where the pCO2 is normal and the
bicarbonate remains excessively elevated (since reversal of renal compensation and a change in bicarbonate retention will not occur

immediately), i.e. the appearances of an acute metabolic alkalosis. Unfortunately, what most often happens is that the pCO2 falls below
normal and an acute mixed respiratory and metabolic alkalosis occurs. Given that both of the acid-base abnormalities in this setting are
pointing in the same direction, a very dangerous elevation in arterial pH can easily occur and can be life-threatening. This is often
referred to as a "post-hypercapnic alkalosis". When placing patients like this on mechanical ventilation it is important to select settings
that allow for a slow fall in pCO2, and checking ABGs frequently to monitor this. End-tidal carbon dioxide monitoring can also be helpful
here.

A 24 year old woman with no significant medical history comes to clinic for routine annual checkup. She states that she has been been
under stress at work and her only complaint is new onset fatigue in the last month that is made worse with exertion and improved by
periods of rest. Her physical examination and routine lab testing is within normal limits. A chest x-ray taken several months ago shows
widening of the mediastinum.
Which of the following would correctly describe her acid-base status?
1)
2)
3)
4)
5)

pH = 7.42, PCO2 = 46, PO2 = 86


pH = 7.18, PCO2 = 19, PO2 = 98
pH = 7.32, PCO2 = 26, PO2 = 66
pH = 7.12, PCO2 = 86, PO2 = 66
pH = 7.56, PCO2 = 56, PO2 = 86

The correct answer is : D

Myasthenia can be a difficult diagnosis, as the symptoms can be subtle and hard to distinguish. The hallmark of myasthenia gravis is
fatiguability. Muscles become progressively weaker during periods of activity and improve after periods of rest. Muscles that control eye
and eyelid movement, facial expression, chewing, talking, and swallowing are especially susceptible. The muscles that control breathing
and neck and limb movements can also be affected. Often the physical examination is within normal limits.

In most cases, the first noticeable symptom is weakness of the eye muscles. In others, difficulty in swallowing and slurred speech may
be the first signs. The degree of muscle weakness involved in myasthenia varies greatly among patients, ranging from a localized form,
limited to eye muscles, to a severe or generalized form in which many muscles - sometimes including those that control breathing - are
affected such as the skeletal muscles of the chest. Subsequently, patients can develop signs and symptoms of acute respiratory

acidosis. Acute respiratory acidosis occurs when an abrupt failure of ventilation occurs and the PaCO2 is elevated above the upper limit
of the reference range (over 47 mm Hg) with an accompanying acidemia (pH <7.35).

A chest X-ray is frequently performed as it may point towards alternative diagnoses (e.g. Lambert-Eaton due to a lung tumor). It may
also identify widening of the mediastinum suggestive of thymoma, but computed tomography (CT) or magnetic resonance imaging (MRI)
are more sensitive ways to identify thymomas, and are generally done for this reason.

Treatment is by medication and/or surgery. Medication consists mainly of cholinesterase inhibitors to directly improve muscle function
and immunosuppressant drugs to reduce the autoimmune process. Thymectomy is the preferred surgical method to treat myasthenia.
For emergency treatment, plasmapheresis or IVIG can be used as a temporary measure to remove antibodies from the blood
circulation.

pH = 7.42, PCO2 = 46, PO2 = 86 (choice A) is consistent with chronic respiratory acidosis such as patients with COPD and obesity
hypoventilation syndrome (i.e., Pickwickian syndrome). The PaCO2 is elevated above the upper limit of the reference range, with a
normal blood pH (7.35 to 7.45) or near-normal pH secondary to renal compensation and an elevated serum bicarbonate (HCO3 >30
mm Hg).

pH = 7.18, PCO2 = 19, PO2 = 98 (Choice B) is consistent with metabolic acidosis such as diarrhea (non gap acidosis) or the
MUDPILES high anion gap acidosis (methanol intoxication, uremia, diabetic ketoacidosis, para-aldehyde, isoniazid/iron, lactic acid,
ethyleneglycol, salicylates).

pH = 7.32, PCO2 = 26, PO2 = 66 (choice C) is consistent with respiratory alkalosis. Respiratory alkalosis may be produced
iatrogenically during excessive mechanical ventilation. Other causes include psychiatric (anxiety and hysteria); CNS (stroke,
subarachnoid haemorrhage, meningitis); pulmonary, such as pulmonary embolism; high altitude areas, where the low atmospheric
pressure of oxygen stimulates increased ventilation; fever, which stimulates the respiratory centre in the brainstem; and pregnancy.

pH = 7.56, PCO2 = 56, PO2 = 86 (choice E) is consistent with metabolic alkalosis such as patient with persistent vomiting or diuretic

abuse.

A mother rushes her 6 year old daughter into the emergency department after finding her passed out in the garage. She says that she
found her unresponsive, lying face down on the floor of their garage, and when she tried to revive her by giving mouth to mouth, she
noted a sweet taste on her lips.
On physical exam, the patient is unarousable with temperature of 37.2 degrees, BP 115/70, HR 95 and RR of 22. The rest of the exam
is unremarkable.
A stat metabolic panel reveals:

Na: 137
K: 3.6
Cl: 95
HCO3: 17
BUN: 20
Cr: 0.7
Glucose: 102

A urine microscopy reveals envelope-shaped crystals. Which of the following would correctly describe her acid-base status?
1)
2)
3)
4)
5)

metabolic acidosis
metabolic alkalosis
respiratory acidosis
respiratory alkalosis
mixed acid-base disorder

The correct answer is : A

The envelope shaped crystals suggests ethylene glycol poisoning as the cause of the metabolic acidosis. Ethylene glycol is widely used
as an automotive antifreeze. In its pure form, it is an odorless, colorless, syrupy, sweet-tasting liquid.

Metabolic acidosis is a condition that occurs when the body produces too much acid or when the kidneys are not removing enough acid
from the body. If unchecked, metabolic acidosis leads to acidemia, i.e., blood pH is low (less than 7.35) due to increased production of
hydrogen by the body or the inability of the body to form bicarbonate (HCO3-) in the kidney. Its causes are diverse, and its
consequences can be serious, including coma and death

Symptoms are aspecific, and diagnosis can be difficult unless the patient presents with clear indications for arterial blood gas sampling.
Symptoms may include chest pain, palpitations, headache, altered mental status such as severe anxiety due to hypoxia, decreased
visual acuity, nausea, vomiting, abdominal pain, altered appetite (either loss of or increased) and weight loss (longer term), muscle
weakness and bone pains. Those in metabolic acidosis may exhibit deep, rapid breathing called Kussmaul respirations which is
classically associated with diabetic ketoacidosis. Rapid deep breaths increase the amount of carbon dioxide exhaled, thus lowering the
serum carbon dioxide levels, resulting in some degree of compensation. Over compensation via respiratory alkalosis to form an
alkalemia does not occur.

Arterial blood gas sampling is essential for the diagnosis. The pH is low (under 7.35) and the bicarbonate levels are decreased (<24
mmol/l). Due to respiratory compensation (hyperventilation), carbon dioxide is decreased and conversely oxygen is increased.

To distinguish between the main types of metabolic acidosis, a clinical tool called the anion gap is considered very useful. It is calculated
by subtracting the chloride and bicarbonate levels from the sodium.

Anion gap = ( [Na+] ) - ( [Cl-]+[HCO3-] )

As sodium is the main extracellular cation, and chloride and bicarbonate are the main anions, the result should reflect the remaining
anions. Normally, this concentration is about 8-16 mmol/l (124). An elevated anion gap (i.e. > 16 mmol/l) can indicate particular types
of metabolic acidosis, particularly certain poisons, lactate acidosis and ketoacidosis.

Metabolic alkalosis (Choice B) would result from decreased hydrogen ion concentration (vomitting or excess aldosterone), leading to

increased bicarbonate, or alternatively a direct result of increased bicarbonate concentrations (hypokalemia or use of alkalotic agents in
peptic ulcer disease). Generally, metabolic alkalosis can be divided into two categories, depending upon urine chloride levels: chlorideresponsive (<10 mEq/L) or Chloride-resistant (>20 mEq/L) metabolic alkalosis.

Respiratory acidosis (choice C) would result from decreased respiration (hypoventilation) and increased PaCO2 (hypercapnia).
Respiratory acidosis can result from problems with the lung (COPD or ARDS) or problems outside the lung that impairs respiration as in
the case of neuromuscular disorders (e.g. Myasthenia gravis).

Respiratory alkalosis (choice D) would result from increased respiration (hyperventilation) that elevates the blood pH. Respiratory
alkalosis may be produced iatrogenically during excessive mechanical ventilation. Other causes include psychiatric (anxiety and
hysteria); CNS (stroke, subarachnoid haemorrhage, meningitis); pulmonary, such as pulmonary embolism; high altitude areas, where
the low atmospheric pressure of oxygen stimulates increased ventilation; fever, which stimulates the respiratory centre in the brainstem;
and pregnancy.

Mixed acid-base disorder (choice E) would result from cases such as salicylate poisoning. Salicylates directly stimulate the respiratory
center to cause hyperventilation (respiratory alkalosis) which is dose-dependent. Metabolic acidosis is the most serious acid-base
disorder and is due to increased production of endogenous acids rather than the salicylate itself. The combination of metabolic acidosis
and respiratory alkalosis can be a difficult situation to diagnose from the blood gases.

A 7 year old boy is brought to the ER in a comatose state. EMS found him on the floor of the bathroom unresponsive with an empty
bottle of pills that his grandmother normally takes. An ABG is obtained showing pH = 7.27, pCO2 = 3.0, pO2 = 14.3, HCO3- = 16.2,
Base Excess = -7.4. A stat metabolic panel is obtained showing Na+ = 143, K+ = 4.5, Cl- = 105, BUN = 12.4, and Creatinine = 4.3.
What is the patient's anion gap?
1)
2)
3)
4)
5)

21.8
13.3
12.1
9.7
2.13

The correct answer is : A

Anion gap is a calculated measurement that is representative of the unmeasured ions in plasma or serum. The 'measured' cations are
sodium (Na+), Potassium (K+), Calcium (Ca2+) and Magnesium (Mg2+). The 'unmeasured' cations include a few normally occurring
serum proteins, and some pathological proteins (e.g., paraproteins found in multiple myeloma). Likewise, the 'measured' anions are
chloride (Cl), bicarbonate (HCO3) and phosphate (H2PO4), with the 'unmeasured' anions being sulphates and a number of serum
proteins (predominantly albumin). By convention only Na+, Cl and HCO3 are used for calculation of the anion gap: ( [Na+] ) (
[Cl]+[HCO3] ).

The average anion gap for healthy adults is 8-12 mEq/L. Anion gap can be classified as either high, normal or, in rare cases, low. A high
anion gap indicates that there is loss of HCO3 without a concurrent increase in Cl. Electroneutrality is maintained by the elevated
levels of anions like lactate, beta-hydroxybutyrate and acetoacetate, PO4, and SO4. These anions are not part of the anion-gap
calculation and therefore a high anion gap results. Thus, the presence of a high anion gap should result in a search for conditions that
lead to an excess of these substances.

The patients anion gap is (143 - (105+16.2))= 21.8. Children often ingest iron pills and consequently develop sever metabolic acidosis
as in the case of this scenario. Knowing that iron is a cause of high anion acidodsis can help eliminate a number of answer choices
quickly.

Choice B (13.3) is consistent with a normal anion gap. In patients with a normal anion gap the drop in HCO3 is compensated for almost
completely by an increase in Cl and hence is also known as hyperchloremic acidosis. The HCO3 lost is replaced by a chloride anion,
and thus there is a normal anion gap. Some common causes are gastrointestinal loss of HCO3 (i.e., diarrhea), renal loss of HCO3
(i.e. proximal renal tubular acidosis also known as type II RTA), renal dysfunction (i.e. distal renal tubular acidosis also known as type I
RTA), ammonium chloride or acetazolamide ingestion, hyperalimentation fluids (i.e. total parenteral nutrition), mineralocorticoid
deficiency (Addison's disease).

Choice C (12.1) is consistent with with a normal anion gap. In patients with a normal anion gap the drop in HCO3 is compensated for
almost completely by an increase in Cl and hence is also known as hyperchloremic acidosis. The HCO3 lost is replaced by a chloride

anion, and thus there is a normal anion gap. Some common causes are gastrointestinal loss of HCO3 (i.e., diarrhea), renal loss of
HCO3 (i.e. proximal renal tubular acidosis also known as type II RTA), renal dysfunction (i.e. distal renal tubular acidosis also known
as type I RTA), ammonium chloride or acetazolamide ingestion, hyperalimentation fluids (i.e. total parenteral nutrition), mineralocorticoid
deficiency (Addison's disease).

D. 9.7 (choice D) is consistent with a normal anion gap. In patients with a normal anion gap the drop in HCO3 is compensated for
almost completely by an increase in Cl and hence is also known as hyperchloremic acidosis. The HCO3 lost is replaced by a chloride
anion, and thus there is a normal anion gap. Some common causes are gastrointestinal loss of HCO3 (i.e., diarrhea), renal loss of
HCO3 (i.e. proximal renal tubular acidosis also known as type II RTA), renal dysfunction (i.e. distal renal tubular acidosis also known
as type I RTA), ammonium chloride or acetazolamide ingestion, hyperalimentation fluids (i.e. total parenteral nutrition), mineralocorticoid
deficiency (Addison's disease).

2.13 (choice E) is consistent with low anion gap. A low anion gap is largely associated with hypoalbuminemia. Albumin is a negatively
charged protein and its loss from the serum results in the retention of other negatively charged ions such as chloride and bicarbonate.
As bicarbonate and chloride anions are used to calculate the anion gap, there is a subsequent decrease in the gap. In
hypoalbuminaemia the anion gap is reduced from between 2.5 to 3 mmol/L per g/dL decrease in serum albumin. Common conditions
that reduce serum albumin in the clinical setting are hemorrhage, nephrotic syndrome, intestinal obstruction and liver cirrhosis.

The mother of one of your patients calls your office requesting antibiotics for her 8 year old daughter. One of the child's classmates was
recently diagnosed with meningococcal meningitis and remains hospitalized at a nearby facility. Your patient sat 2 seats behind the this
child in class in the days before the illness, and the two frequently played together on the playground. The mother reports that her
daughter remains afebrile and well-appearing currently.
Which of the following is the most appropriate management of this patient?
1)
2)
3)
4)
5)

Reassurance and follow up only if symptoms develop


Immediate office visit and physical examination
Provide telephone prescription for ciprofloxacin
Schedule office visit for one week
Lumbar puncture and intravenous ceftriaxone

The correct answer is : A.

Following exposure to a patient with Neisseria meningitides, antibiotic prophylaxis is recommended - if the exposure was close or
significant. Thus, the American Academy of Pediatrics recommends providing prophylaxis to patients who are close household
contacts or who have had close personal contact (kissing, sharing toothbrushes, etc.) to an affected patient within the preceding 7 days.
Prophylaxis should also be provided to caregivers who provided mouth-to-mouth resuscitation or performed endotracheal intubation.
Younger children in daycare settings are another special circumstance where prophylaxis is recommended.

Prophylaxis can be provided with a short course of oral rifampin, a single intramuscular dose of ceftriaxone, or (for older patients) a
single dose of ciprofloxacin.

For the patient in the vignette, who does not meet any of these criteria, reassuring the mother that the child is extremely unlikely to
develop meningococcal infection is appropriate, and no chemoprophylaxis is necessary. However, if the child should become febrile or
manifest any other symptoms of illness, she should be evaluated.

An immediate office visit and physical examination (choice B) is not necessary. In this circumstance, even the most experienced
clinician is unlikely to detect any abnormalities on physical examination that would change the child's management. Meningococcal
meningitis is not a subtle diagnosis, and any of the signs of this illness should be noticed by an observant caregiver. An office visit may
be useful in the event that the patient's mother is not sufficiently reassured that her child is at very low risk of infection, however.

Providing a prescription via telephone (choice C) would be appropriate if the child had a significant exposure meeting criteria for
prophylaxis; no physical examination would be required in that case. However, using a fluoroquinolone in a patient of this age would be
a bad choice, given the concern for joint toxicity. A different agent should be used.

Scheduling an office visit one week from now (choice D) will not be helpful. If the patient required prophylaxis, the "window of

opportunity" would likely have closed by that point.

Performing a lumbar puncture and administering ceftriaxone (choice E) is inappropriate in a patient who has no clinical evidence of
meningitis.

A 14 year old male presents with a localized area of painful swelling on his lower abdomen. On examination, you find a 14 x 20 cm area
of warmth and erythema, with a central area measuring 5 x 4 cm with induration and fluctuance. An incision and drainage is performed,
and yielding approximately 5 mL of thick, yellowish purulent material. Culture of the expressed material growsStaphylococcus aureus,
and rapid antibiotic sensitivity testing shows the following antimicrobial susceptibility results and interpretations:

Ampicillin - Resistant
Ampicillin-sulbactam - Resistant
Cefazolin - Resistant
Cephalexin - Resistant
Clindamycin - Sensitive
Doxycycline - Resistant
Erythromycin - Resistant
Nafcillin - Resistant
Oxacillin - Resistant
Linezolid - Sensitive
Trimethoprim-sulfamethoxazole - Sensitive
Vancomycin - Sensitive

Which of the following is the most appropriate oral antibiotic to treat this patient's cellulitis?
1)
2)
3)
4)
5)

Cephalexin
Clindamycin
Dicloxacillin
Trimethoprim-sulfamethoxazole
Vancomycin

The correct answer is : D.

This patient has a skin infection caused by methicillin-resistant Staphylococcus aureus (MRSA). MRSA is an isolate of S. aureus that is
highly resistant to most antibiotics, including all beta-lactams. (Since methicillin is seldom used in clinical practice, resistance to
oxacillin, nafcillin, and dicloxacillin imply methicillin resistance, and the term MRSA is still used.)

Most skin and soft tissue MRSA infections are caused by a particular strain of MRSA that fortunately retains some susceptibility to older
classes of antibiotics such as clindamycin, doxycycline, and trimethoprim-sulfamethoxazole. However, many strains that initially appear
sensitive to clindamycin in vitro will fail when used in vivo due to the presence of an inducibile resistance gene. This particular
resistance pattern is suggested by resistance to erythromycin, as the gene that permits constitutive resistance to erythromycin is often
transmitted with the gene for inducible resistance to clindamycin.

Whenever an MRSA isolate is listed as resistant to erythromycin, a D-test should be performed to ensure that inducible resistance to
clindamycin is not present prior to treatment. This involves culturing the isolate on a dish containing antibiotic eluting disc of both
clindamycin and erythromycin. In the presence of erythromycin, the MSRA isolate will demonstrate its resistance sooner, leading to a
blunting of the central area of clearing on the side nearest the erythromycin disc (making a "D" shape).

Cephalexin (choice A) resistance can be presumed based on the fact that this is MRSA - resistance to one beta-lactam implies
resistance to all of the others.

Clindamycin (choice B) could be successful in this situation - or treatment may fail for the reasons indicated above. It is thus not the
best oral antibiotic choice.

Dicloxacillin (choice C) is an anti-staphylococcal penicillin, and would be useful if this were not a MRSA isolate.

Vancomycin (choice E) is not absorbed from the GI tract, and thus must always be administered parenterally to treat systemic
infections. For the treatment of infections that are localized to the GI tract - such as Clostridium difficile, for example - oral vancomycin
can be effective.

A 58 year old man comes to your office for medical clearance for an elective cholecystectomy. After reviewing his chart, he has history
of multiple myeloma, iron deficiency anemia from lower GI bleeding, nephrotic syndrome, and delirium tremens from alcohol withdrawal.
Physical exam reveals a thin cachectic man with clubbing, palmar erythema, spider nevi, and a number of veins on his abdomen.
All the following is consistent with his low anion gap except?
1)
2)
3)
4)
5)

chronic liver disease


nephrotic syndrome
multiple myeloma
iron poisoning
poor nutritional status

The correct answer is : D

A low anion gap is largely associated with hypoalbuminemia. Albumin is a negatively charged protein and its loss from the serum results
in the retention of other negatively charged ions such as chloride and bicarbonate. As bicarbonate and chloride anions are used to
calculate the anion gap, there is a subsequent decrease in the gap. In hypoalbuminaemia the anion gap is reduced from between 2.5 to
3 mmol/L per g/dL decrease in serum albumin. Common conditions that reduce serum albumin in the clinical setting are hemorrhage,
nephrotic syndrome, intestinal obstruction and liver cirrhosis.

Albumin is a major protein in the human body, making up about 60% of total human plasma protein by mass. Many hormones, drugs,
and other molecules are mostly bound to albumin in the bloodstream and must be released before becoming biologically active. For
example, calcium binds to albumin and hypoalbuminemia leads to an increase in free ionized calcium.

chronic liver disease (choice A) can lead to hypoalbuminemia. Albumin is synthesized in the liver, and low serum albumin may be
indicative of liver failure or diseases such as cirrhosis or chronic hepatitis. This particular patient has a number of physical findings that

are consistent with the stigmata of chronic liver disease due to hyperestrogenism such as clubbing, palmar erythema, spider nevi
(angiomata), gynaecomastia, feminizing hair distribution, testicular atrophy.

nephrotic syndrome (choice B) is characterized by the loss of albumin in urine due to kidney damage. Patients often develop lower
extremity edema or anasarca due to a disturbance in starling forces (i.e. decrease in oncotic pressure).

multiple myeloma (choice C) leads to a low anion gap due to an increase in plasma IgG (paraproteinaemia). This should not be
confused with hyperalbuminaemia, as an elevation in albumin concentration causes an increased anion gap.

poor nutritional status (choice E) can lead to low albumin levels. Levels below 3.5 grams per deciliter are generally consistent with
hypoalbuminemia due to chronic malnutrition.

Several hours after attending a picnic at a family reunion, a 10 year old male develops fever to 39.2 C, accompanied by abdominal
pain. On examination, there is marked tenderness to palpation with rebound tenderness and voluntary guarding over the right
abdomen, especially in the right lower quadrant. Both the obturator and Rovsing's signs are present. An appendectomy is
performed, and inflammation of the terminal ileum is noted intraoperatively. Subsequent pathologic analysis shows a non-inflammed,
normal appendix.
Which of the following is the most likely organism causing this patient's presentation?
1)
2)
3)
4)
5)

Bacillus cereus
Campylobacter jejuni
Escherichia coli O157:H7
Salmonella typhi
Yersinia enterocolitica

The correct answer is : E.

This patient has pseudoappendicitis, most commonly caused by Yersinia species (Y. enterocolitica or Y. pseudotuberculosis). The
organism leads to inflammation of the terminal ileum that can mimic the classic anatomical location of pain in patients with appendicitis.
Had a CT scan been performed before surgery in this vignette, the diagnosis might have been suggested by the appearance of terminal
ileitis and mesenteric adenitis. Treatment is primarily supportive, as antibiotic therapy does not appear to alter the natural history of the
disease.

Bacillus cereus (choice A) produces an exotoxin that causes acute gastroenteritis, typically after the consumption of improperly cooked
or stored rice or other grains.

Campylobacter jejuni (choice B) causes foodborne gastroenteritis typically caused by consumption of contaminated poultry. After
infection has occurred, Guillain-Barre syndrome may develop.

Escherichia coli O157:H7 (choice C) causes bloody diarrhea, and may lead to the development of hemolytic-uremic syndrome in
children.

Salmonella typhi (choice D) causes typhoid fever, which causes high fevers, vomiting and nonbloody diarrhea, and a rash of flat, rose
colored spots.

You are seeing a newborn male infant born at 38 weeks gestation to a 24 year old mother with human immunodeficiency virus (HIV)
infection. She received antiretrovirals throughout the pregnancy, and the infant was delivered by elective Cesarean section. The
infant's Apgar scores were 8 and 9; physical examination is within normal limits.
Which of the following is the most appropriate next step in the management of this patient?
1)
2)
3)
4)
5)

Encourage breastfeeding and standard well newborn care


Obtain enzyme-linked immunosorbent assay (ELISA) for HIV
Begin zidovudine anti-retroviral therapy immediately
Check absolute CD4 lymphocyte count
Referral to social services

6) All of the above are correct

The correct answer is : C.

In the modern era, maternal-infant transmission of HIV is very rare when appropriate precautions are taken. Among these are
continuation of retrovirals during pregnancy (to keep the maternal viral load low) and elective Cesarean section (to prevent trauma and
transmission of bodily fluids during vaginal delivery).

Another extremely important step is the prompt initiation of zidovudine (AZT) to the infant, as soon as possible after birth. This
medication should be continued until it is definitively documented (by at least two separate tests) that the infant does not have HIV.

Encouraging breastfeeding (choice A) is inappropriate. Although the risk of HIV transmission through breastmilk is low, especially when
mothers are receiving HAART, current recommendations in developed countries (where safe alternatives to breastfeeding are readily
available) are to encourage strict avoidance of breastfeeding to limit infection transmission risk.

Obtaining an ELISA for HIV (choice B) will not be helpful. Remember, the ELISA is an antibody test for HIV. You would expect that the
infant's ELISA would be positive, because he has received maternal antibody through normal transplacental transmission. To diagnose
the presence of HIV in the infant would require a direct test for the HIV genome.

Checking a CD4 lymphocyte count (choice D) will not be helpful. Even if the infant were infected with HIV, you would anticipate that
their CD4 count would still be normal.

A referral to social services (choice E) is not medically required based on the information presented in the vignette. Many women have
HIV; the mere presence of maternal HIV infection does not necessarily mean that the mother is an incompetent caregiver or in need of
social services.

An 18 month old male is brought to his pediatrician for evaluation of purulent drainage from his left eye. He had been well until 3 days
prior, when he developed upper respiratory congestion and cough. This morning, he had fever to 39.5 C, and his eye attained the
appearance shown below. On examination, you note injection of the left conjunctiva, especially prominent temporally, as well as mild
eyelid edema and mucopurulent drainage. There are a few 1 cm lymph nodes in the anterior cervical chain, and yellow mucus in the
nares bilaterally. The right tympanic membrane is opalescent and mobile; the left is slightly bulging and hyperemic, with a thin crescent
of pus inferiorly. The lung fields are clear to auscultation, and the mucus membranes are moist.

Which of the following is the best treatment for this patient's disorder?
1)
2)
3)
4)
5)

Warm compresses and tear duct massage


Ophthalmic moxifloxacin
Oral amoxicillin
Oral amoxicillin-clavulanate
Intramuscular benzathine penicillin

The correct answer is : D.

This patient has otitis-conjunctivitis syndrome, characterized by unilateral conjunctivitis with ipsilateral otitis media. This is caused by
nontypable Haemophilus influenzae, an organism that possesses beta-lactamases; the best treatment, therefore, is a beta-lactamase

stable antibiotic such as amoxicillin-clavulante.

Warm compresses and tear duct massage (choice A) are appropriate therapies for a nasolacrimal duct obstruction, a common cause of
purulent eye drainage in infants.

Ophthalmic moxifloxacin (choice B) provides good broad spectrum antibacterial coverage against most bacterial causes of
conjunctivitis; it would not treat the patient's otitis media.

Oral amoxicillin (choice C) is not generally effective against Haemophilus influenzae species, as it is degraded by beta-lactamases.

Intramuscular benzathine penicillin G (choice E) is a good treatment for acute group A streptococcal pharyngitis, as it eradicates
streptococci and requires only a single dose. It would not be effective against a beta-lactamase producing organism like H. influenzae.

A 24 year old woman with a history of recurrent diverticulitis is hospitalized after a three day history of LLQ abdominal pain. At the time
of admission, she is afebrile and her vitals her 132/76, RR 18, P98, and O2 saturation of 99% on room air. A CT of the abdomen and
pelvis shows a phlegmon in her sigmoid colon and she is placed on IV Zosyn and made NPO. On her second hospital day, she
develops persistent diarrhea and fever and her stool is sent for clostridium difficile toxin. A repeat set of vitals show a temperature of
39.6C, BP 142/76, RR 28, P98, and O2 saturation of 99% on room air.
Which of the following would correctly describe her primary acid-base disturbance?
1)
2)
3)
4)
5)

Low pH, Low HCO3, High Base Deficit, Low pCO2.


Low pH, High HCO3, High Base Excess, High pCO2.
Neutal pH, Low HCO3, High Base Deficit, Low pCO2.
High pH, High HCO3, High Base Excess, High pCO2.
High pH, Low HCO3, High Base Excess, Low pCO2.

The correct answer is : A.

Base excess is defined as the amount of strong acid that must be added to each liter of fully oxygenated blood to return the pH to 7.40
at a temperature of 37C and a pCO2 of 40 mmHg. Comparison of the base excess with the reference range assists in determining
whether an acid/base disturbance is caused by a respiratory, metabolic, or mixed metabolic/respiratory problem. While carbon dioxide
defines the respiratory component of acid-base balance, base excess defines the metabolic component.

The predominant base contributing to base excess is bicarbonate. Thus, a deviation of serum bicarbonate from the reference range is
ordinarily mirrored by a deviation in base excess. However, base excess is a more comprehensive measurement, encompassing all
metabolic contributions.

The patient has diverticulitis complicated by phlegmon and is placed on IV antibiotics. Subsequently, she develops signs of clostridium
difficile colitis. C. difficile colitis is a common nosocomial infection that results in persistent diarrhea from IV antibiotics, which kills the
normal gut flora allowing C. difficile to proliferate and to produce toxin . Diarrhea has large amounts of excreted bicarbonate, resulting in
a nongap metabolic acidosis.

Low pH, High HCO3, High Base Excess, High pCO2 (Choice B) is consistent with respiratory acidosis such as inadeqaute aveolar
ventilaltion (ARDS or Myasthenia gravis)

Neutal pH, Low HCO3, High Base Deficit, Low pCO2 (choice C) is consistent with a mixed metabolic/respiratory problem such as
salicylate poisoning.

High pH, High HCO3, High Base Excess, High pCO2 (choice D) is consistent metabolic alkalosis such as excessive vomitting and
Cushings disease.

High pH, Low HCO3, High Base Excess, Low pCO2 (Choice E) is consistent with respiratory alkalosis as in hyperventilated states (e.g.
pulmonary embolism)

A 7 year old girl develops the acute onset of fever and throat pain. Her past history is notable for mild persistent asthma and allergic
rhinitis. Current medications include cetirizine and inhaled fluticasone; she has no medication allergies. On examination, the girl is
mildly ill appearing. The tonsils are enlarged and covered with a thick yellow-white exudate. There are numerous tender anterior
cervical lymph nodes, and a diffuse, erythematous sandpapery rash concentrated in the flexor skin creases. You obtain a culture of the
posterior oropharynx; results will be reported by the laboratory in two days.
Which of the following is the most appropriate treatment for this patient's illness?
1)
2)
3)
4)
5)

Analgesics and supportive care


Azithromycin
Levofloxacin
Penicillin
Trimethoprim-sulfamethoxazole

The correct answer is : D.

This is a classic case of strep throat, or pharyngitis caused by group A beta-hemolytic Streptococcus pyogenes(GAS).

All of the key diagnostic features are present: sudden onset of fever and sore throat, absence of symptoms (like runny nose or cough)
that suggest a viral infection, tender cervical lymphadenopathy, exudative pharngitis, and the "sandpapery" rash of scarlet fever. While
the culture will establish the diagnosis definitively, the features are classic enough here that treatment should be provided empirically to
prevent the development of post-streptococcal complications like rheumatic fever or peritonsillar abscess.

In an era of increasing antibiotic resistance, Streptococcus pyogenes has remained exquisitely sensitive to penicillin, and this drug
remains the gold standard for treatment of GAS infections in non-allergic patients. A course of oral penicillin or a single intramuscular

injection of penicillin can be given and are equally efficacious.

Analgesics and supportive care (choice A) would be appropriate therapy for a viral pharngitis. However, the scarlatiniform rash is highly
specific for GAS infection; antimicrobial therapy should be provided to prevent rheumatic fever and suppurative complications like
peritonsillar abscess.

Azithromycin (choice B) has reliable activity against GAS, but does not provide as rapid killing as penicillin.

Levofloxacin (choice C) does have good activity against GAS. However, due to concerns about possible joint/tendon toxicity,
fluoroquinolones are generally avoided in pediatric patients unless there is a compelling reason for their use.

Trimethoprim-sulfamethoxazole (choice E) is useful for some strains of GAS, but others are resistant; it should thus be avoided not only
for streptococcal pharyngitis but in other syndromes in which GAS are likely etiologic causes (such as cellulitis).

You are asked to see a 22-year-old morbidly obese patient in the emergency room. He has been feeling generally unwell for the last 3-4
days. On further questioning he admits to adhering to a 600 calorie diet and a 2000 ml water intake daily for the last 10 days.
Which of the following is the most likely primary acid-base disorder to develop in this clinical setting?
1)
2)
3)
4)
5)

metabolic acidosis, normal anion gap


metabolic acidosis, increased anion gap
acute respiratory alkalosis
no acid-base disorder is likely to develop
metabolic alkalosis

The correct answer is : B.

Our patient is morbidly obese with a prolonged hypocaloric intake. The most likely abnormality in this setting is ketoacidosis induced by

fasting, and complicated by the potential for our patient to have pre-existing insulin resistance, non-insulin dependent diabetes, and
hyperlipidemia.

The metabolic milieu associated with morbid obesity can amplify the acid-base effects of fasting significantly. Traditionally, fasting
ketoacidosis has been considered to a mild abnormality, usually associated with a ketoacid level < 10 mmol/L and a serum bicarbonate
of 13-16 mmol/L. In this setting pH may fall to around 7.3.

In the normal state, fasting will deplete liver glycogen stores within 18-26 hours, during which time the body increases the process of
gluconeogenesis (formation of new glucose molecules from lactate, pyruvate, muscle amino acids etc.), and also supplies energy to
tissues as ketone bodies. Insulin levels (although low) and response remain sufficient to suppress severe lipolysis and excessive
transfer of free fatty acids to the liver for conversion to ketone bodies. Ketoacid generation is maximal after 3-14 days of fasting.

In the morbidly obese patient, insulin levels may be absolutely or relatively deficient (insulin resistance) in prolonged fasting, and
accompanied by elevated glucagon levels. The inhibiting effect of insulin on lipolysis is removed, promoting significant free fatty acid
release and subsequent uptake by the liver. Free fatty acid metabolism results in increased acetyl-coA production. In the liver, this
substance is either utilized in FFA production (which will be turned off here by the low insulin level) or ketone body formation. The
metabolic state of this patient is such that he will be producing significant amounts of ketoacids including acetone, acetoacetone, and
beta-hydroxybutyrate. Although some of these will be renally excreted, sufficient levels will remain to cause systemic acidosis.

Ketoacidosis is an anion-gap metabolic acidosis. It is possible that this patient will lower his pH sufficiently to stimulate a compensatory
respiratory alkalosis, but this is not the primary abnormality.

Fasting ketoacidosis is important, because it can occur during hospital stay, and unless considered in the differential diagnosis can lead
to unnecessary investigation for other causes of acidosis. It often occurs in patients with non-insulin dependent diabetes after major
surgery if insulin and dextrose are not provided. You will also see fasting ketoacidosis go hand in hand with alcoholic ketoacidosis. In
this case, alcohol intake can lead to increases acetyl-coA generation and so a shift to ketone generation.

Ketone body detection with Acetest uses nitroprusside to react with the acetone and acetoacetate. It must be remembered that in
cases where beta-hydroxybutyrate is the major ketoacid (and alcoholic ketoacidosis may be such a case) this test can be artificially
negative or underestimate the level of ketoacid present.

A 6 week old female infant is brought to her pediatrician for evaluation of tachypnea. Her mother reports that the infant has had a dry
cough for the past three weeks that seems to be getting worse. She denies fever or any other symptoms. Vital signs include
temperature 36.8 C, pulse 132 bpm, respirations 50/min, and blood pressure 80/50. On examination, you hear soft rales throughout the
lung fields. A CBC shows a WBC 10,500 with a differential of 41% lymphocytes, 35% neutrophils, 19% eosinophils, 4% monocytes, and
1% basophils. A chest x-ray demonstrates hyperinflation and interstitial infiltrates.
Which of the following is the most likely organism causing this infant's respiratory illness?
1)
2)
3)
4)
5)

Bordatella pertussis
Chlamydia trachomatis
Listeria monocytogenes
Respiratory syncitial virus
Streptococcus pneumoniae

The correct answer is : B.

Chlamydia trachomatis causes a distinctive type of pneumonia in infants - one that is very commonly tested on standardized medical
exams. Affected patients are typically 2-8 weeks old and present with tachypnea and a chronic or smoldering "staccato" cough, though
they are afebrile. Physical examination reveals crackles, and x-rays indicate interstitial infiltrates. Peripheral eosinophilia is also
common. Treatment is with macrolides such as azithromycin or erythromycin.

Bordatella pertussis (choice A) causes pertussis or "whooping cough." Fits of repetitive, forceful coughing are typical. A common clue
to diagnosis is the presence of a marked absolute lymphocytosis (often over 20,000).

Listeria monocytogenes (choice C) can cause a fulminant pneumonia in infants. The organism can cross the placenta; infection is often
foodborne from unpasteurized products or improper refrigeration.

Respiratory syncitial virus (choice D) is the most common cause of bronchiolitis. The key feature on physical examination is the
presence of wheezing.

Streptococcus pneumoniae (choice E) is the most common cause of lobar pneumonia in all age groups. Patients are typically febrile
and ill-appearing.

A 68 year old man is brought to the ER by EMS after being found unresponsive by his wife. He underwent a radical resection of his
bladder for bladder carcinoma a month ago and has been experiencing persistent diarrhea since his discharge two weeks prior. His only
other medical issues are glaucoma (treated with acetazolamide) and previous myocardial infarction but a complete history was not
available.
On physical exam, the patient is unarousable even with sternal rubs and the rest of the exam is unremarkable. His vitals are
temperature of 37.2 degrees, BP 115/70, HR 95 and RR of 22. A stat metabolic panel reveals:

Na: 137
K: 3.6
Cl: 95
HCO3: 20
BUN: 60
Cr: 3.4
Glucose: 342

Which of the following would explain the patients metabolic acidosis?


1) treatment of glaucoma
2) radical surgery of urinary bladder cancer
3) Persistent diarrhea

4) renal tubular acidosis type IV


5) diabetic ketoacidosis

The correct answer is : E

To distinguish between the main types of metabolic acidosis, a clinical tool called the anion gap is considered very useful. It is calculated
by subtracting the chloride and bicarbonate levels from the sodium.

Anion gap = ( [Na+] ) - ( [Cl-]+[HCO3-] )

As sodium is the main extracellular cation, and chloride and bicarbonate are the main anions, the result should reflect the remaining
anions. Normally, this concentration is about 8-16 mmol/l (124). An elevated anion gap (i.e. > 16 mmol/l) can indicate particular types
of metabolic acidosis, particularly certain poisons, lactate acidosis and ketoacidosis.
Caluclating this patients anion gap reveals a high anion gap, which shortens the differential diagnosis considerably.

Ketoacidosis is a type of metabolic acidosis associated with high concentrations of ketone bodies, formed by the breakdown of fatty
acids and the deamination of amino acids. The two common ketones produced in humans are acetoacetic acid and -hydroxybutyrate.
In diabetic patients, ketoacidosis is usually accompanied by insulin deficiency, hyperglycemia, and dehydration. Particularly in type 1
diabetics the lack of insulin in the bloodstream prevents glucose absorption and can cause unchecked ketone body production (through
fatty acid metabolism) potentially leading to dangerous glucose and ketone levels in the blood. Hyperglycemia results in glucose
overloading the kidneys and spilling into the urine (transport maximum for glucose is exceeded). Dehydration results following the
osmotic movement of water into urine (Osmotic diuresis), exacerbating the acidosis.

Treatment of glaucoma (choice A) with acetazolamide can lead to normal anion gap acidosis. Acetazolamide is a carbonic anhydrase
inhibitor, which forces the kidneys to excrete bicarbonate (HCO3-), thus re-acidifying the blood.

Radical surgery of urinary bladder cancer (choice B) can lead to a normal anion gap acidosis from ureteral diversion. Urine from the
ureter may be diverted to the sigmoid colon due to disease (uretero-colonic fistula) or after bladder surgery. subsequently, urinary Cl- is
absorbed by the colonic mucosa in exchange for HCO3- and increase the gastrointestinal loss of HCO3-.

Persistent diarrhea (choice C) leads to a non-gap acidosis from a loss of GI secretions which are rich in bicarbonate.

Renal tubular acidosis type IV (choice D) leads to a non-gap acidosis from dysfunctional renal tubular cells that causes an inappropriate
wastage of HCO3- and retention of Cl-.

A 57-year-old male with COPD, home oxygen dependence, exercise tolerance of 20-30 yards, and daily chronic sputum production, is
on maintenance treatment with several types of metered dose inhalers and no other medications. He presents to your clinical with a 12
hour history of purulent sputum production, and 3-4 hours of worsening shortness of breath increased weakness. When you see him he
appears to be in moderate respiratory distress with peripheral cyanosis and mild confusion.
Which of the following is the most likely acid-base disorder to be present at this time?
1)
2)
3)
4)
5)

chronic respiratory alkalosis


acute respiratory acidosis
acute respiratory acidosis and chronic respiratory acidosis
acute respiratory alkalosis
no acid-base disorder is likely to develop

The correct answer is : C.

This man has an acute exacerbation of severe COPD. Given his baseline functional impairment it is highly likely that he has developed
chronic CO2 retention. Over the course of time his body will have compensated for this respiratory acidosis by renal retention of
bicarbonate. The effectiveness of this metabolic compensation for a respiratory acidosis depends on the duration of the acidosis, and on
normal renal handling of bicarbonate.

The elevated CO2 level is dealt with acutely through the action of carbonic anhydrase inside red blood cells: CO2 + H2O <-> H2CO3 <->
H+ + HCO3-. The H+ generated in this process is then buffered by intracellular protein or phosphates. In acute respiratory acidosis there
is no time for renal compensation and so the [HCO3-] rises by only by around 1 mmol/L for every 10mmHg increase in pCO2 above its
reference value of 40mmHg.

Acute CO2 retention stimulates ventilation via both central and peripheral chemoreceptors and causes cerebral vasodilation and
sympathetic nervous system activation> This may contribute to agitation and confusion, but consciousness will become depressed as
pCO2 continues to rise. In a patient who is already subject to maximum renal bicarbonate retention due to chronic CO2 retention, any
acute exacerbation of this can only be handled by buffering, and so will be limited in success and extent.

The renal response is at a maximum after 2-4 days, although it is already evident by 6-14 hours. At its peak, there is a 4 mmol/L
increase in [HCO3-] for every 10mmHg increase in pCO2 from the reference value of 40mmHg.

Our patient will have a chronic (compensated) respiratory acidosis which is now accompanied by a new acute respiratory acidosis.

A 17 year old girl is brought to the emergency room for respiratory difficulty and shortness of breath. According to the patients mother,
she has been coughing green sputum and vomiting for the past three days and has been taking aspirin for fever. She has a history of
moderate persistent asthma and has been taking inhaled corticosteroids.
On physical examination, she is lethargic with dry mucous membranes and a jugular venous pressure of less than 5 cm. Auscultation

reveals crackles at her lung bases. She becomes hyperreflexive and complains of diffuse muscle aches in a tremulous voice. She
develops stridor and becomes apneic, which leads to emergent intubation and mechanical ventilation.
Which of the following would explain her hyperreflexia?
1)
2)
3)
4)
5)

Steroid toxicity
Salicylate poisoning
Hypoventilation
Lobar pneumonia
Protracted vomiting

The correct answer is : E.

Severe vomiting leads to loss of hydrogen and chloride ions and the development of a hypochloremic contraction alkalosis. This results
from a loss of water in the extracellular space which is poor in bicarbonate. Since water is lost while bicarbonate is retained, the
increased concentration of bicarbonate "mops up" more of the hydrogen ions and raises the blood pH.

One of the consequences of alkalosis is the reduction in the ionized serum calcium and the development of tetany. Tetany is
characterized by sensory symptoms consisting of paresthesias (odd feelings) of the lips, tongue, fingers and feet; carpopedal spasm,
which may be prolonged and painful; generalized muscle aching; and hyperreflexia.

Tetany characteristically is considered to result from a severe degree of hypocalcemia (literally, very low calcium levels in the blood). It
can also result from reduction in the ionized fraction of plasma calcium without marked hypocalcemia, as occurs in severe alkalosis.
Although the calcium that is typically measured in laboratory medicine is the calcium in blood plasma, what really causes tetany is low
ionic calcium in the extracellular fluid (between cells) and intracellular fluid (within cells).

Steroid toxicity (choice A) would not cause a primary acid base disturbance.

Salicylate poisoning (choice B) is one of the causes of a high anion gap metabolic acidosis. The causes of high anion gap metabolic
acidosis can be summarized with mnemonic MUDPILES (methanol intoxication, uremia, diabetic ketoacidosis, para-aldehyde,
isoniazid/iron, lactic acid, ethyleneglycol, salicylates).

Hypoventillation (Choice C) would lead to retention of carbon dioxide (CO2) and result in a respiratory acidosis.

Lobar pneumonia (choice D) would not cause a primary acid base disturbance.

A 30-year-old woman known to have type 1 diabetes presents to the emergency room with a productive cough, fever, and polyuria for 4
days and progressive lethargy and confusion for the last day. She complains of abdominal pain and nausea with vomiting. Her breath
smells fruity, her serum glucose is 600 mg/dL, and her serum potassium is 5.5. The patient is started on IV fluids, insulin, and
potassium.
Which of the following metabolites would be elevated in the patients blood?
1)
2)
3)
4)
5)

Formic acid
Lactic acid
Acetoacetic acid and beta hydroxybutyrate
Glycolic acid
Phosphoric acid and sulphuric acid

The correct answer is : C

Ketoacidosis is a type of metabolic acidosis associated with high concentrations of ketone bodies, formed by the breakdown of fatty
acids and the deamination of amino acids. The two common ketones produced in humans are acetoacetic acid and -hydroxybutyrate.
Ketoacidosis is a pathological metabolic state marked by extreme and uncontrolled ketosis. Normal ketosis, by contrast, is a functional
aspect of fat-based energy metabolism, induced by prolonged fasting or a low-carbohydrate diet. In ketoacidosis, the body fails to
adequately regulate ketone production causing such a severe accumulation of keto acids that the pH of the blood is substantially
decreased. In extreme cases ketoacidosis can be fatal.

Ketoacidosis is most common in untreated type 1 diabetes mellitus, when the liver breaks down fat and proteins in response to a
perceived need for respiratory substrate. Prolonged alcoholism may lead to alcoholic ketoacidosis. Fasting leads to ketosis but not
ketoacidosis.
Ketoacidosis can be smelled on a person's breath. This is due to acetone, a direct byproduct of the spontaneous decomposition of
acetoacetic acid. It is often described as smelling like fruit or nail polish remover.

In diabetic patients, ketoacidosis is usually accompanied by insulin deficiency, hyperglycemia, and dehydration. Particularly in type 1
diabetics the lack of insulin in the bloodstream prevents glucose absorption and can cause unchecked ketone body production (through
fatty acid metabolism) potentially leading to dangerous glucose and ketone levels in the blood. Hyperglycemia results in glucose
overloading the kidneys and spilling into the urine (transport maximum for glucose is exceeded). Dehydration results following the
osmotic movement of water into urine (osmotic diuresis), exacerbating the acidosis.

Formic acid (choice A) results from methanol poisoning and can cause permanent blindness by destruction of the optic nerve. The initial
symptoms of methanol intoxication include central nervous system depression, headache, dizziness, nausea, lack of coordination,
confusion, and with sufficiently large doses, unconsciousness and death. Methanol poisoning can be treated with the antidotes ethanol
or fomepizole.

Lactic acid (choice B) results from lactic acidosis and is considered a distinct form of high anion gap metabolic acidosis. The condition
typically occurs when tissues receive too little oxygen (hypoxia or hypoperfusion) and cells are forced to metabolize glucose
anaerobically. Therefore, excess pyruvate produced is converted into lactate and released into the bloodstream. Lactic acidosis is
characterized by lactate levels >5 mmol/L and serum pH <7.35.

Glycolic acid (choice D) is produced when ethanol is metabolized by the enzyme alcohol dehydrogenase (ADH) pathway, which is
located in the liver and gastric mucosa, and by the cytochrome P-450 mixed function oxidase (MFO) system in the liver. As with ethyl
alcohol and methanol, ethylene glycol is metabolized by alcohol dehydrogenase to form glycoaldehyde. Through interaction with
aldehyde dehydrogenase, ethylene glycol is then metabolized to glycolic acid. Glycolic acid and its toxic byproducts first affect the
central nervous system, then the heart, and finally the kidneys. A profound acidosis often ensues and is attributable to the glycolic acid
in circulation. The patient may develop hyperventilation and Kussmaul respirations that results from high anion gap acidemia.

Phosphoric acid and sulphuric acid (choice E) results from chronic renal failure. If the renal damage affects both glomeruli and tubules,
the acidosis is a high-anion gap acidosis. It is due to failure of adequate excretion of various acid anions due to the greatly reduced
number of functioning nephrons. If the renal damage predominantly affects the tubules with minimal glomerular damage, a different type
of acidosis may occur. This is called Renal Tubular Acidosis (RTA) and this is a normal anion gap or hyperchloraemic type of acidosis.
The GFR may be normal or only minimally affected.

A 21 year-old female presents to your office complaining of itchy eyes for the last two months. She has no other past medical history.
Her vitals are as follows: blood pressure 110/70, heart rate 70 bpm, temperature 98.8F, respiratory rate 16. The patient is a well
nourished woman with a BMI of 26. On head and neck exam, she has significant ocular dryness and conjunctival injection, cracking of
her lips and parotid enlargement. Her cardiac, pulmonary and abdominal exam are unremarkable.
Which of the following would correctly describe her acid-base status?
1)
2)
3)
4)
5)

metabolic acidosis
metabolic alkalosis
respiratory acidosis
respiratory alkalosis
mixed acid-base disorder

The correct answer is : B

The patient has signs of bulimia nervosa that is characterized by recurrent binge eating, followed by compensatory behaviors. The most
common form is defensive vomiting, fasting, and the use of laxatives, enemas, diuretics. Physical exam often reveals severe dental
carries caused by frequent contact between teeth and gastric acid, calluses or scars on back of hands due to repeated trauma from
incisors, and swollen parotid glands.

Bulemic patients can develop Chloride-responsive (<10 mEq/L) metabolic alkalosis due to multiple reasons. In chloride-responsive
metabolic alkalosis, alkalosis can result from loss of hydrogen ions from either vomitting (direct loss of hydrogen and chloride ions with
stomach contents) or renal loss of hydrogen ions from increased sodium-hydrogen exchange protein in the kidney (Conns syndrome).

Additionally, a contraction alkalosis can develop from from a loss of water in the extracellular space which is poor in bicarbonate,
typically from diuretic use. Since water is lost while bicarbonate is retained, the increased concentration of bicarbonate "mops up" more
of the hydrogen ions and raises the blood pH. Hydration with normal saline corrects chloride-responsive metabolic alkalosis.

Metabolic acidosis (choice A) would arise in situations such as diarrhea (non gap acidosis) or the MUDPILES high anion gap acidosis
(methanol intoxication, uremia, diabetic ketoacidosis, para-aldehyde, isoniazid/iron, lactic acid, ethyleneglycol, salicylates).

Respiratory acidosis (choice C) would result from decreased respiration (hypoventilation) and increased PaCO2 (hypercapnia).
Respiratory acidosis can result from problems with the lung (COPD or ARDS) or problems outside the lung that impairs respiration as in
the case of neuromuscular disorders (e.g. Myasthenia gravis).

Respiratory alkalosis (choice D) would result from increased respiration (hyperventilation) that elevates the blood pH. Respiratory
alkalosis may be produced iatrogenically during excessive mechanical ventilation. Other causes include psychiatric (anxiety and
hysteria); CNS (stroke, subarachnoid haemorrhage, meningitis); pulmonary, such as pulmonary embolism; high altitude areas, where
the low atmospheric pressure of oxygen stimulates increased ventilation; fever, which stimulates the respiratory centre in the brainstem;
and pregnancy.

Mixed acid-base disorder (choice E) would result from cases such as salicylate poisoning. Salicylates directly stimulate the respiratory
center to cause hyperventilation (respiratory alkalosis) which is dose-dependent. Metabolic acidosis is the most serious acid-base
disorder and is due to increased production of endogenous acids rather than the salicylate itself. The combination of metabolic acidosis
and respiratory alkalosis can be a difficult situation to diagnose from the blood gases.

A 34 year old diabetic woman was brought to the emergency department. She suffered from an attack of gastroenteritis and had
vomiting and diarrhea for the past few days. As a result she missed her insulin doses for two consecutive days. There was no dysuria
and no evidence of chest, pelvic or skin infection. At the time of admission, her blood gas analysis showed the following:

Na+: 130 (normal 136-146 mmol/L)


Cl- : 80 (normal 95-106 mmol/L)
HCO3- : 10 (22-29 mmol/L)

pH: 7.21
pCO2: 25
Glucose: 250mg/dl

Which of the following is a correct analysis of her condition?


1)
2)
3)
4)
5)
6)

Anion gap metabolic acidosis


Severe diabetic ketoacidosis
Hyperosmolar hyperglycemic state (HHS)
A and B
B and C
A, B and C

The correct answer is : D.

For analysis of any acid base disorder, the following six steps are helpful:

1. Whether there is an acidemia or alkalemia? If the pH < 7.35, acidemia (and at least 1 acidosis) is present. If the pH > 7.45,
alkalemia (and at least 1 alkalosis) is present. For this case, severe acidosis must be present.

2. Is the primary process metabolic or respiratory? pH < 7.35 and pCO2 < 40 is a metabolic acidosis. If the pH < 7.35 and pCO2
> 40 is a respiratory acidosis. pH > 7.45 and pCO2 < 40 is a respiratory alkalosis. If pH > 7.45 and pCO2 > 40 is a metabolic
acidosis.

3. If the primary process is respiratory, is it acute or chronic?

4. Is there an anion gap (AG)? To calculate anion gap, Na+ - Cl- - HCO3- > 12, which in this case is 130-80-10=40. Thus there is
an anion gap. Anion gap metabolic acidosis (choice A) is present.

5. Is the respiratory compensation adequate? For metabolic acidosis, pCO2~1.5(HCO3-)+8= 21-25 normal range, in this case
1.5(10)+8=23, thus respiratory compensation is normal.

6. Are there any other metabolic disturbances? Corrected HCO3- = Measured HCO3- + Normal AG-12. Thus: 10+(40-12)=
10+28=38, since this overcorrects, a secondary metabolic disorder is present.

The common causes of high anion gap are methanol, uremia, diabetic ketoacidosis, paraldehyde, isopropyl alcohol. Ketoacidosis
(choice B) is a common complication in patients with type 1 diabetes (may also occur in those with type 2 diabetes under certain
circumstances) due to poor insulin therapy compliance or intercurrent illness such as seen in this patient. Ketone bodies in the urine
should confirm this diagnosis.

Hyperosmolar hyperglycemic state (HHS) (choice C) is another serious metabolic derangement that occurs in patients with diabetes
mellitus and can be a life-threatening emergency. The condition is characterized by hyperglycemia, hyperosmolarity, and dehydration
without significant ketoacidosis. HHS usually presents in older patients with type 2 diabetes mellitus and carries a higher mortality rate
than DKA. Most patients present with severe dehydration and focal or global neurologic deficits. In as many as one third of cases, the
clinical features of HHS and DKA overlap and are observed simultaneously (overlap cases).

Based on the consensus statement published by the American Diabetic Association, diagnostic features of HHS may include the
following:

Plasma glucose level of 600 mg/dL or greater

Effective serum osmolality of 320 mOsm/kg or greater

Profound dehydration up to an average of 9L

Serum pH greater than 7.30

Bicarbonate concentration greater than 15 mmol/L

Small ketonuria and absent-to-low ketonemia

Some alteration in consciousness

Our patient does not have the above symptoms; hence choice C is not applicable in this case.

A mother brings her 18 month old son for evaluation of possible immunodeficiency. The boy was born at 38 weeks' gestation weighing
3440g and was well for the first four months of his life, but following entry to daycare he has been "sick all the time." His typical
symptoms include congestion, rhinorrhea, cough, and intermittent fever; he has also had two episodes of acute otitis media that were
treated successfully with amoxicillin. He has never been hospitalized. The family history is benign, and the social history is notable for
cigarette smoke exposure in the home. On examination, you note a blonde, fair skinned interactive male toddler whose height is 85 cm
(78th percentile) and weight is 12.7 kg (75%). No specific abnormalities are found on examination.
Which of the following is the most appropriate next step in the management of this patient?

1)
2)
3)
4)
5)

Sweat chloride test


Fluorescent in-situ hybridization (FISH) for chromosome 22q11.2 deletion
Nitroblue-tetrazolium test
Serum IgA, IgG, IgE, and IgM levels
Reassurance

The correct answer is : E.

Previous studies have shown that the average child experiences 4-5 respiratory infections per year, and that many normal children
experienced up to 12 per year. Given that even a common cold lasts 10-14 days in young children (whose immune systems are
naieve), it is clearly understandable that a parent would feel that their child is "always sick." Children who attend daycare or who are
exposed to tobacco smoke experience greater numbers of infections early in life.

The fact that this patient's examination is normal and he is growing well is very reassuring that he does not have a specific
immunodeficiency. Also extremely important is the nature of his recurrent infections - recurrent URIs and otitis media do not suggest
immunodeficiency in the same manner that recurrent invasive meningococcal infections or life-threatening Candida infections would,
since a certain number of these infections are expected among patients in this age group.

A sweat chloride test (choice A) would screen for cystic fibrosis, which could present with frequent respiratory tract infections. However
- especially on standardized medical exams - patients will often have other features of cystic fibrosis, such as poor growth, nasal polyps,
malabsorptive diarrhea, or delayed passage of meconium.

FISH for chromosome 22q11.2 deletion or DiGeorge syndrome (choice B) is unlikely to be helpful. Patients with DiGeorge do have
impaired T-cell activity and can get recurrent infections; however, none of the other features of this disorder were provided in the
question stem (high arched palate, conotruncal heart defects, hypocalcemia, etc.).

The nitroblue-tetrazolium test (choice C) could diagnose chronic granulomatous disease. Affected patients get recurrent or lifethreatening infections with catalase-positive organisms like S. aureus, Serratia marcescens,Aspergillus, and Nocardia.

Serum immunoglobulin levels (choice D) would help diagnose a specific antibody deficiency.

A 36 year old woman presents to her primary care doctor for medical clearance. She is planning on a elective hernial repair. She was
recently diagnosed with sarcoidosis several months ago and placed on prednisone. Upon inspection, her physician notices a well
nourished woman with telangiectasias and purple red marks on her extremeties. She also has some signs of hirsutism that he did not
notice on her previous visit several months ago.
Which of the following would correctly describe her primary acid-base disturbance?
1)
2)
3)
4)
5)

Low pH, Low HCO3, High Base Deficit, Low pCO2.


Low pH, High HCO3, High Base Excess, High pCO2.
Neutal pH, Low HCO3, High Base Deficit, Low pCO2.
High pH, High HCO3, High Base Excess, High pCO2.
High pH, Low HCO3, High Base Excess, Low pCO2.

The correct answer is : D.

Base excess is defined as the amount of strong acid that must be added to each liter of fully oxygenated blood to return the pH to 7.40
at a temperature of 37C and a pCO2 of 40 mmHg. Comparison of the base excess with the reference range assists in determining
whether an acid/base disturbance is caused by a respiratory, metabolic, or mixed metabolic/respiratory problem. While carbon dioxide
defines the respiratory component of acid-base balance, base excess defines the metabolic component.

The predominant base contributing to base excess is bicarbonate. Thus, a deviation of serum bicarbonate from the reference range is
ordinarily mirrored by a deviation in base excess. However, base excess is a more comprehensive measurement, encompassing all
metabolic contributions.

The woman is exhibiting signs of iatrogenic Cushing's disease with chronic steroid use. Symptoms include rapid weight gain, particularly
of the trunk and face with sparing of the limbs (central obesity). A common sign is the growth of fat pads along the collar bone and on
the back of the neck (buffalo hump) and a round face often referred to as a "moon face". Other symptoms include hyperhidrosis (excess

sweating), telangiectasia (dilation of capillaries), thinning of the skin (which causes easy bruising and dryness, particularly the hands)
and other mucous membranes, purple or red striae on the trunk, buttocks, arms, legs or breasts, proximal muscle weakness (hips,
shoulders), and hirsutism (facial male-pattern hair growth), baldness and/or cause hair to become extremely dry and brittle.

The excess corticosteroids have some mineralocorticoid effects, thus causing renal overproduction of bicarbonate The metabolic
alkalosis is most severe with the syndrome of ectopic ACTH production from adrenal adenomas.

Low pH, Low HCO3, High Base Deficit, Low pCO2 (choice A) is consistent with metabolic acidosis such as diarrhea (non gap acidosis)
or the MUDPILES high anion gap acidosis (methanol intoxication, uremia, diabetic ketoacidosis, para-aldehyde,
isoniazid/iron, lactic acid, ethyleneglycol, salicylates).

Low pH, High HCO3, High Base Excess, High pCO2 (Choice B) is consistent with respiratory acidosis such as inadeqaute aveolar
ventilaltion (ARDS or Myasthenia gravis)

Neutal pH, Low HCO3, High Base Deficit, Low pCO2 (choice C) is consistent with a mixed metabolic/respiratory problem such as
salicylate poisoning.

High pH, Low HCO3, High Base Excess, Low pCO2 (Choice E) is consistent with respiratory alkalosis as in hyperventilated states (e.g.
pulmonary embolism)

You see a 57-year-old male for routine review at your clinic. He has a 10 year history of COPD, and has been dependent on home
oxygen for the last 5 years. He has daily chronic sputum production and can only walk slowly for 20-50 yards at a time. He is on
treatment with several types of metered dose inhalers and no other medications, and his condition is stable at present. Which of the
following is the most likely acid-base disorder to be present?
1) chronic respiratory alkalosis
2) acute respiratory acidosis

3) chronic respiratory acidosis


4) no acid-base disorder is likely to develop
5) mixed acid-base disorder

The correct answer is : C.

This man has advanced lung disease, with loss of functioning alveolar units and an increase in respiratory dead space. This will reduce
the removal of CO2 from the blood via the lungs, which in turn will lead to a respiratory acidosis. This will have developed slowly, and as
the PaCO2 climbed the body will have compensated for the respiratory acidosis by renal retention of bicarbonate.

The change in bicarbonate level with PaCO2 differs when the respiratory acidosis is chronic rather than acute. In an acute respiratory
acidosis the bicarbonate level increases 1 mmol/L for each 10 mmHg rise in PaCO2, and a significant proportion of the buffering occurs
intracellularly by proteins and phophates. In a chronic respiratory acidosis the compensation is much more effective, with a rise in
bicarbonate of 3.5-5 mmol/L for each 10 mm Hg rise in PaCO2. This renal response is maximal by 3-5 days.

It was thought that maximal compensation for a chronic respiratory acidosis was never enough to fully to fully return the pH to normal,
and that a normal pH in this setting indicated a concomitant metabolic alkalosis. This has been challenged and studies have found
normal pH in chronic stable patients with no risk factors for metabolic alkalosis. Compensation in these patients was equivalent to an
increase in bicarbonate of 5.1 mmol/L for every 10 mmHg increase in PaCO2.

Four days ago, a 4 year old male developed fever to 39.8 C, and has refused to bear weight on his right leg for the past 3 days. On
examination, you note mild swelling of the right proximal tibia. Overlying the medial aspect of the tibia, 8 cm below the patella, is a 4 x 4
cm area of warmth and erythema with exquisite tenderness to palpation. Range of motion at the knee, hip, and ankle is normal.
Laboratory evaluation shows a WBC of 14,800/mcL with 82% polymorphonuclear lymphocytes. An erythrocyte sedimentation rate is 80
mm/h. You obtain plain film x-rays of the right femur, tibia, and fibula which show no fracture or bony defect.
Which of the following is the most appropriate next step in the management of this patient?
1) Obtain blood culture

2)
3)
4)
5)

Measure C-reactive protein (CRP)


Begin vancomycin
Begin gentamicin
Referral to an orthopedic surgeon

The correct answer is : A.

This patient has acute osteomyelitis, as evidenced by his body pain and point tenderness and elevated laboratory markers of
inflammation.

The diagnosis should be confirmed radiographically; while plain film x-rays are abnormal in many cases, bony lucencies are often a late
finding, and thus MRI is preferable when standard x-rays are unrevealing.

Most cases of acute osteomyelitis in normal hosts are caused by Staphylococcus aureus. However, it is extremely important to obtain
an etiologic diagnosis whenever possible, as knowledge of the infecting organism and its antibiotic susceptibilities will help determine
the treatment provided. Blood cultures are frequently positive in cases of osteomyelitis; when they are not, a bone biopsy sent for
culture may yield the causative organism.

In some other cases of infectious illnesses that can present with refusal to bear weight - such as septic arthritis - providing antibiotic
therapy immediately is of paramount importance. For patients with osteomyelitis, however, there is no such urgency, and antibiotics
should not be given until appropriate culture material has been obtained. (If this patient's examination had been concerning for joint
involvement, then joint aspiration or emergent radiographic studies would have been appropriate.)

Measurement of the patient's C-reactive protein (choice B) will not provide any more clinically useful information than you already have.
Clinically, the suspicion for osteomyelitis is high - you need to establish that diagnosis definitively (and etiologically if possible), and
begin treatment. While CRP values may be useful to follow over time to monitor response to therapy, it is not the best choice at this
point.

Vancomycin (choice C) covers almost all gram positive organisms including Staphylococcus aureus, and would be a good choice - once
appropriate cultures have been obtained.

Gentamicin (choice D) covers gram negatives well, and when given with a cell-wall active agent, can provide synergy in staphylococcal
killing. However, beginning antibiotics at this point is not the best answer among those listed.

Referral to an orthopedic surgeon (choice E) is not the best choice at this point. Surgery may be required if a periosteal abscess is
present, or if tissue is needed for culture to establish an etiologic diagnosis. However, you should work up the patient first. Referral to a
specialist is seldom the correct response on standardized medical examinations, as question authors want you to know what to do with
a patient; you should only select this response if all of the appropriate workup and therapy have already been completed or if the
situation is emergent.

A 9 month old male infant has a fever to 40.5 C, then has a tonic-clonic seizure. The child was born at term and was previously entirely
well, though he has not received any childhood immunizations due to parental religious beliefs. On examination, you note a lethargic, illappearing infant who withdraws to noxious stimuli. A lumbar puncture is performed, and analysis of the cerebrospinal fluid shows the
following:

Appearance: Yellow, turbid


WBC Elevated (98% polymorphonuclear cells)
RBC 18
Protein 106 mg/dL
Glucose 9 mg/dL
Gram stain: many WBCs; few gram positive cocci in pairs

Which of the following is the most appropriate antibiotic therapy for this patient?
1)
2)
3)
4)
5)

Ceftriaxone
Tobramycin
Piperacillin-tazobactam
Ampicillin and gentamicin
Cefotaxime and vancomycin

The correct answer is : E.

This patient has bacterial meningitis. His gram stain suggests that the causative organism is Streptococcus pneumoniae.

In many areas, strains of S. pneumoniae that are highly resistant to penicillins have become common. In most sites in the body,
this resistance can be overcome by using higher doses of the drug. However, in the cerebrospinal fluid, even when high drug doses are
used in the presence of inflamed meninges, adequate drug levels to overcome the penicillin resistance cannot be obtained. Therefore,
vancomycin should be used until organism sensitivities are known. However, because vancomycin is a bacteriostatic drug, a thirdgeneration cephalosporin should also be used for more rapid organism killing if the organism is not highly resistant. The best choice,
then, is cefotaxime and vancomycin (choice E).

Ceftriaxone (choice A) and other third-generation cephalosporins penetrate the CSF well and have good broad spectrum activity against
gram positives and gram negatives, including most of the common causes of meningitis. The only flaw with their use in this situation is
penicillin-resistant S. pneumoniae; vancomycin should be added empirically until sensitivities are known.

Tobramycin (choice B) and other aminoglycosides have excellent gram negative activity. However, they penetrate non-inflammed
meninges poorly, and would be inappropriate monotherapy for meningitis.

Piperacillin-tazobactam (choice C) has excellent activity against gram negatives (including Pseudomonas) and anaerobes. However, it
will not provide any added benefit against a highly penicillin resistant S. pneumoniaestrain.

Ampicillin and gentamicin (choice D) display synergy when used together, leading to rapid killing. This would be a good regimen for
a neonate with meningitis, where gram positive cocci in the CSF would represent group B streptococcus, not Streptococcus

pneumoniae.

A 2 year old male develops fever to 41 C, along with nausea and vomiting. The patient was previously well, had developed normally,
and received all immunizations. On examination, you find a somnolent, ill-appearing toddler with rapid, deep respirations. The patient is
extremely irritable when you attempt to perform the physical examination, especially with manipulation of his neck. A lumbar puncture is
performed, and the following results are obtained from the patient's cerebrospinal fluid.

White blood count 4250/mcL (85% polymorphonuclear cells, 15% mononuclear cells)
Red blood count 842/mcL
Protein 112 mg/dL
Glucose 18 mg/dL

Which of the following are the most likely organisms to be isolated from this patient's cerebrospinal fluid culture?
1)
2)
3)
4)
5)

Escherichia coli and Streptococcus agalactiae


Neisseria meningitides and Streptococcus pneumoniae
Neisseria meningitides and Staphylococcus aureus
Haemophilus influenzae type b and Streptococcus pneumoniae
Haemophilus influenzae type b and Streptococcus agalactiae

The correct answer is : B.

The most common agents causing meningitis varies across pediatric age groups, but knowing the common culprits for patients of
different ages is critically important, as the empiric antibiotics selected will depend on the likely causative organisms in that clinical
situation. (For this reason, questions about this topic are extremely common on standardized medical examinations!).

For children 2 years old and above, the most common pathogen is Neisseria meningitides, followed byStreptococcus pneumoniae. The
incidence of the latter has dramatically declined following the successful introduction of a pneumococcal vaccine. (Even more
successful has been the vaccine for Haemophilus influenzaetype b, which has nearly eradicated this once common cause of childhood
meningitis.)

Escherichia coli and Streptococcus agalactiae or group B strep (choice A) are the most common organisms causing meningitis in the
neonatal period.

While Neisseria meningitides is the most common organism in this age group, Staphylococcus aureus is a very uncommon cause of
meningitis generally, and typically occurs only following surgical instrumentation or when there is an anatomical defect (such as a dural
sinus tract) allows this organism access to the meninges.

While Streptococcus pneumoniae remains fairly common, Haemophilus influenza type b (choice D) has nearly been eradicated by
successful vaccination programs in developed countries.

As noted previously, Haemophilus influenzae type b is very uncommon in the era of vaccination, whileStreptococcus agalactiae is very
uncommon outside of the newborn period.

A 50-year-old male with a history of diabetes develops sudden onset shortness of breath on the general surgery floor. His current drug
regimen includes glipizide, metformin, and metprolol. A pulmonary embolulus is suspected and he is sent for a CT angiogram with IV
contrast. Two days later, he develops deep and rapid breathing and abdominal pain.
Which of the following is the likely cause of his lactic acidosis type B?
1)
2)
3)
4)
5)

septic shock
severe anemia
metformin
thiamine deficiency
respiratory failure

The correct answer is : C.

Metformin is an oral anti-diabetic drug in the biguanide class. It is the first-line drug of choice for the treatment of type 2 diabetes,

particularly in overweight and obese people and those with normal kidney function.

When prescribed appropriately, metformin causes few adverse effect. Unlike many other anti-diabetic drugs, it does not cause
hypoglycemia if used alone. Lactic acidosis can be a serious concern in overdose and when it is prescribed to people with
contraindications. Metformin is contraindicated in people with any condition that could increase the risk of lactic acidosis, including
kidney disorders, lung disease and liver disease. It is recommended that metformin be temporarily discontinued before any radiographic
study involving iodinated contrast (such as a contrast-enhanced CT scan or angiogram), as contrast dye may temporarily impair kidney
function, indirectly leading to lactic acidosis by causing retention of metformin in the body. It is recommended that metformin be
resumed after two days, assuming kidney function is normal

Lactic acidosis is a physiological condition characterized by low pH in body tissues and blood accompanied by the buildup of lactate,
and is considered a distinct form of high anion gap metabolic acidosis. The condition typically occurs when tissues receive too little
oxygen (hypoxia or hypoperfusion) and cells are forced to metabolize glucose anaerobically. Therefore, excess pyruvate produced is
converted into lactate and released into the bloodstream. Lactic acidosis is characterized by lactate levels >5 mmol/L and serum pH
<7.35.

Signs of lactic acidosis are deep and rapid breathing (Kussmaul breathing), vomiting, and abdominal painsymptoms that may easily
be mistaken for other problems. Lactic acidosis may be caused by diabetic ketoacidosis or kidney and liver disease. Heavy metals
(arsenic and cyanide) and medications such as metformin and anti-HIV drugs (antiretrovirals) can also cause lactic acidosis by
mitochondrial toxicity.

The Cohen-Woods classification categorizes causes of lactic acidosis as follows:

Type A: Decreased perfusion or oxygenation

Overproduction: Circulatory, pulmonary, or hemoglobin transfer disorders are commonly responsible.

Underutilization: Liver disease, gluconeogenesis inhibition, thiamine deficiency, and uncoupled oxidative phosphorylation

Type B: Evidence of poor tissue perfusion or oxygenation is absent

B1: Systemic disease such as renal and hepatic failure, diabetes and malignancy.

B2: Medication or intoxication (biguanides, alcohols, iron, isoniazid, zidovudine and salicylates)

B3: Inborn error of metabolism

Septic shock(choice A) decreases tissue perfusion and oxygen delivery as a result of infection and sepsis, though the microbe maybe
localized to a particular site.

Severe anemia (choice B) decreases tissue perfusion and oxygen delivery as a result of the decreased oxygen carrying capability of the
blood. There may be signs of a hyperdynamic circulation: a fast heart rate (tachycardia), flow murmurs, and cardiac enlargement as the
body tries to compensate with increased cardiac output.

Thiamine deficiency (choice D) leads to impaired aerobic respiration and is often prevalent in patients with impaired nutrition (chronic
alcoholism, malaborption syndromes, HIV-AIDS, parenteral glucose or hyperalimentation without adequate B-vitamin supplementation).

Respiratory failure (choice E) decreases tissue perfusion and oxygen delivery due to inadequate gas exchange by the respiratory
system. Therefore, arterial oxygen and/or carbon dioxide levels cannot be maintained within their normal ranges. A drop in blood
oxygenation is known as hypoxemia; a rise in arterial carbon dioxide levels is called hypercapnia. The normal reference values are:
oxygen PaO2 greater than 60 mmHg , and carbon dioxide PaCO2 less than 45 mmHg.

You might also like